Sie sind auf Seite 1von 28

Test 1ª vuelta distancia

Pediatría

Test 1ª vuelta distancia

Pediatría
1. Señale la afirmación correcta respecto al test de Apgar:
5. ¿Cuál de los siguientes fenómenos dermatológicos que podemos
1) Se debe practicar únicamente a los RN patológicos o supuesta- encontrar en un neonato NO debe sugerirnos una patología
mente patológicos. subyacente?:
2) Entre los parámetros que se valoran se encuentra la frecuencia
respiratoria. 1) Manchas hipopigmentadas geográficas.
3) Se debe realizar a los 1-5-10 minutos de vida. 2) Ictericia precoz en las primeras 24 horas.
4) Un test de Apgar de 4 al minuto de vida implica un alto riesgo de 3) Nevus flammeus.
parálisis cerebral. 4) Melanosis pustulosa.
5) La palidez cutánea generalizada supone una puntuación de 1 en 5) Manchas café con leche.
el test de Apgar.
6. Ante un neonato a término que tras cesárea presenta precozmente
2. RN que, al minuto de vida, presenta FC de 110 latidos por taquipnea y cianosis que desaparece con pequeñas cantidades
minuto, acrocianosis con esfuerzo respiratorio ausente, así como de oxígeno, en el que la auscultación pulmonar es normal y en
hipotonía y leve mueca al introducir la sonda de aspiración. La el que la radiografía muestra marcas vasculares prominentes y
puntuación de Apgar será: líquido en las cisuras sin broncograma aéreo, sospecharemos:

1) 1. 1) Enfermedad de membrana hialina.


2) 2. 2) Taquipnea transitoria del recién nacido.
3) 3. 3) Síndrome de aspiración meconial.
4) 4. 4) Persistencia de la circulación fetal.
5) 5. 5) Síndrome de Wilson-Mikity.

3. ¿Cuál de las siguientes afirmaciones sobre los cuidados del 7. RN de 35 semanas de edad gestacional que en las primeras 12
recién nacido prematuro es FALSA?: horas de vida, presenta taquipnea, quejido audible y cianosis
progresiva, refractaria a la administración de oxígeno. A la
1) La alimentación debe iniciarse siempre por sonda nasogástrica auscultación encontramos crepitantes, sobre todo en bases
en los menores de 32 semanas de edad gestacional, por la falta pulmonares. En la gasometría aparece hipoxemia progresiva,
de coordinación entre succión, deglución y respiración. aumento de la PCO2 y acidosis metabólica. ¿Qué radiografía de
2) En los casos que necesiten tratamiento con eritropoyetina, debe tórax esperaría encontrar en este momento?:
iniciarse a la vez tratamiento con hierro vía oral.
3) Las necesidades de líquidos son mayores que en los neonatos a 1) Normal.
término, tanto más cuanto más prematuro sea el recién nacido. 2) Infiltrado de patrón reticulogranular con broncograma aéreo.
4) Debe aumentarse el intervalo de dosis de todos los medicamentos 3) Hiperinsuflación y aumento de las marcas vasculares, sin bronco-
de eliminación renal. grama.
5) Hay que realizar en todos ecografía transfontanelar aunque no 4) Patrón de esponja.
tengan alteraciones en la exploración neurológica. 5) Borde de ambos pulmones colapsados.

4. Señale la afirmación FALSA respecto a las lesiones producidas 8. En el tratamiento de la EMH se incluyen las siguientes medidas,
por traumatismos obstétricos: EXCEPTO:

1) El cefalohematoma es una hemorragia subperióstica, y está deli- 1) Administración de surfactante endotraqueal.


mitado por las suturas craneales. 2) Antibioterapia.
2) La clavícula es el hueso que se fractura con mayor frecuencia 3) Administración de oxígeno y ventilación mecánica, si es preciso.
durante el parto. 4) Prostaglandinas.
3) El llamado “nódulo de Stroemayer” suele requerir tratamiento 5) Control del aporte de líquidos.
quirúrgico.
4) La rotura hepática no siempre precisa tratamiento quirúrgico. 9. Un niño de 42 semanas de gestación, arrugado, pálido, hipotó-
5) Las fracturas craneales más frecuentes en el parto son las lineales. nico, apneico y cubierto de líquido amniótico verdoso. Respecto

CTO Medicina • C/Nuñez de Balboa 115 • 28006 Madrid • Tfno. (0034) 91 782 43 30/33/34 • E-mail: secretaria@ctomedicina.com • www. ctomedicina.com 1

Willy Aguilar
Test 1ª vuelta distancia
Pediatría
al cuadro que puede padecer este paciente, señale la opción
FALSA: 13. RNPT de 29 semanas de edad gestacional sufre distress respi-
ratorio desde el nacimiento. A las 36 horas de vida presenta
1) La primera maniobra en su reanimación debe ser la aspiración de hipotensión, bradicardia, cianosis y la fontanela se encuentra
tráquea bajo visión laringoscópica. abombada. En el hemograma presenta 15.000 leucocitos, neu-
2) Es un patología típica de RNPT y RNT. trófilos 45%, linfocitos 42%, cayados 0%, plaquetas 180.000,
3) El E. coli es el germen más frecuente de infección bacteriana en hematocrito 30%. ¿Qué prueba complementaria consideraría
estos pacientes. más adecuada en este momento?:
4) En la Rx de tórax es típico encontrar un patrón de atrapamiento
aéreo. 1) Punción lumbar y análisis del LCR.
5) Esta patología produce una llamativa hipertensión pulmonar.
2) Ecocardiograma.
3) Ecografía cerebral.
10. Recién nacido de 38 semanas que tras expulsivo dificultoso 4) TC craneal.
presenta el brazo izquierdo pegado al tronco junto con cierta 5) Rx tórax.
rotación interna de dicho miembro. Señale la respuesta verda-
dera respecto a la lesión obstétrica que sospecha en este recién 14. Un RN pretérmino de 1.800 g de peso, con antecedentes de
nacido:
hipoxia perinatal, a los 10 días de vida presenta distensión
abdominal importante, vómitos y deposiciones hemorrágicas.
1) El reflejo de Moro no estará presente en el lado izquierdo. En la Rx de abdomen se observa edema de asas intestinales,
2) Se debe a una lesión en las raíces inferiores del plexo braquial. con un patrón en “miga de pan” y presencia de gas en la pared
3) Este tipo de parálisis es menos frecuente que la parálisis de intestinal. ¿Cuál sería la opción terapéutica más adecuada en
Klumpke. este paciente?:
4) Es característica la presencia de mano caída.
5) Suele asociarse con síndrome de Horner.
1) Enema de solución hiperosmolar.
2) Alimentación enteral a débito continuo a través de una sonda
11. Un RN de 32 semanas, con antecedente de enfermedad de nasogástrica.
membrana hialina sometido a ventilación mecánica con altas 3) Actitud expectante y vigilancia cuidadosa, ante la posibilidad de
concentraciones de oxígeno, tiene actualmente un mes de vida perforación intestinal.
y sigue dependiendo del respirador para mantener adecuadas
4) Se debe suspender la alimentación enteral y pautar fluidoterapia
saturaciones de oxígeno. En los últimos días presenta además
y antibioterapia i.v.
oliguria con edemas en miembros inferiores y se palpa el hígado
5) La presencia de neumatosis intestinal aconseja la intervención
a 5 cm bajo el reborde costal derecho. En la Rx de tórax se observa
quirúrgica urgente.
una imagen de pequeñas zonas redondas y claras que alternan
con otras de mayor densidad. En relación con la patología que
15. RN de 38 semanas de edad gestacional presenta distensión
presenta este paciente, señale la respuesta FALSA:
abdominal y vómitos biliosos durante el primer día de vida. En
el examen físico se palpan cordones duros que siguen el marco
1) Se consideran factores de riesgo para su desarrollo tener distress
cólico. Una radiografía en bipedestación muestra una masa en
respiratorio severo que precise largos períodos de administración
de O2. pompa de jabón en cuadrante inferior derecho. El tratamiento
2) Las principales causas de muerte de estos pacientes son la bron- inmediato más apropiado es:
quiolitis necrotizante y fallo cardíaco derecho.
3) El tratamiento incluye broncodilatadores, diuréticos y dexameta- 1) Antibioterapia.
sona. 2) Supositorios de glicerina.
4) La nefrolitiasis puede ser una complicación de estos pacientes, 3) Enema de solución hiperosmolar.
secundaria al uso de diuréticos y a la alimentación parenteral. 4) Estimulación rectal.
5) A pesar de que pueda retirarse la administración de O2 antes de salir 5) Cirugía.
de la unidad de cuidados intensivos el pronóstico para la mayoría
de estos pacientes es malo, presentando en la adolescencia obs- 16. Señale la afirmación FALSA de entre las siguientes respecto a
trucción de las vías respiratorias e hiperreactividad bronquial. la ictericia fisiológica:

12. Recién nacido a término que presenta dificultad respiratoria 1) Las cifras máximas de bilirrubina suelen alcanzarse a los 3-4
inmediata, con importante tiraje intercostal y supraesternal, días.
taquipnea, tórax hiperinsuflado y abdomen excavado. En la 2) La ictericia en los RNPT suele ser de inicio algo más tardío y más
auscultación pulmonar destaca abolición casi total del mur- prolongada que en los RNT.
mullo vesicular en el hemitórax izquierdo, con tonos cardíacos 3) Puede comenzar en las primeras 24 horas de vida en el RNT.
desplazados a la derecha. Señale la respuesta verdadera acerca 4) Su duración suele ser inferior a 10-15 días.
de la patología que sospecha: 5) En los RNPT pueden alcanzarse cifras de hasta 14 mg/dl.

1) En más del 50% de los casos hay otras malformaciones aso- 17. En relación a la ictericia por lactancia materna o síndrome de
ciadas. Arias es FALSO que:
2) El tratamiento es quirúrgico, y debe realizarse lo antes posible,
tratando después las alteraciones que haya asociadas. 1) Su frecuencia es aproximadamente de 1 por cada 200 RN a término
3) El diagnóstico diferencial con malformaciones pulmonares quís- que reciben lactancia materna.
ticas se hace con ecografía. 2) Es una ictericia que aparece a la mitad o final de la primera semana,
4) El reflujo gastroesofágico es una secuela rara tras la cirugía. alcanzando su máximo en la tercera.
5) Los afectados suelen desarrollar hipertensión pulmonar a partir del 3) Se pueden alcanzar cifras de hasta 30 mg/dl, de ahí el riesgo de
nacimiento, que en los casos graves condiciona su pronóstico. kernicterus en estos neonatos.

CTO Medicina • C/Nuñez de Balboa 115 • 28006 Madrid • Tfno. (0034) 91 782 43 30/33/34 • E-mail: secretaria@ctomedicina.com • www. ctomedicina.com 2

Willy Aguilar
Test 1ª vuelta distancia
Pediatría
4) En la patogenia se implican varias sustancias presentes en la leche 4) La probabilidad de que el RN presente hipoglucemia es más
materna como pregnanodiol, ácidos grasos de cadena larga y elevada cuanto más altas se encuentren las cifras de glucosa en
glucuronidasas. sangre de cordón.
5) Cuando se suspende la lactancia materna durante 2-4 días se 5) Se trata mediante infusión i.v. de bolos de glucosa y debe retrasarse
produce una rápida disminución de la bilirrubina sérica. la alimentación oral para minimizar el riesgo de hipoglucemia de
rebote.
18. En relación a la enfermedad hemolítica por isosensibilización
ABO, señale la FALSA: 23. NO es propio del RN hijo de madre diabética:

1) La madre suele ser O. 1) Megalencefalia.


2) Los anticuerpos causantes suelen ser de la clase Ig G. 2) Síndrome asfíctico.
3) Pueden resultar afectos los productos del primer embarazo. 3) Hiperbilirrubinemia.
4) El antígeno más frecuentemente implicado es A1. 4) Síndrome de regresión caudal.
5) La enfermedad se agrava en los embarazos siguientes, si no 5) Cardiomiopatía.
hacemos profilaxis.
24. En la sepsis del RN, los gérmenes más frecuentes son:
19. Un lactante de 1 semana es llevado a Urgencias en estado
de coma con hemorragias en la retina y palidez importante. 1) Klebsiella.
Entre sus antecedentes destaca que el parto tuvo lugar en 2) E. coli y estreptococos del grupo A.
el domicilio, no presentó complicaciones posteriores y está 3) E. coli y estreptococos del grupo B.
recibiendo lactancia materna. Señale lo FALSO respecto a 4) Estafilococo.
este cuadro: 5) Gardnerella vaginalis.

1) Se debe a un déficit de los factores dependientes de vitamina K. 25. ¿Qué haría usted ante un RN de madre con hepatitis B activa en
2) Suele iniciarse a partir de las primeras 24 horas. el momento del nacimiento?:
3) Es más frecuente en hijos de madres que han tomado fenitoína o
fenobarbital. 1) Observar la aparición de los síntomas y luego tratar.
4) Las complicaciones hemorrágicas son menos frecuentes en niños 2) Extraer sangre de cordón para estudiar transaminasas.
que toman LM que en los que reciben biberón. 3) Exanguinotransfusión inmediata, si la madre tiene el antígeno
5) Se trata con la administración de vitamina K y/o plasma fresco HBe positivo.
congelado. 4) Poner una dosis bimensual de gammaglobulina.
5) Iniciar vacunación y gammaglobulina en paritorio.
20. Respecto a la anemia en el período neonatal, señale cuál de las
siguientes afirmaciones es FALSA: 26. Un RN de bajo peso para su edad gestacional presenta un
exantema petequial, hepatoesplenomegalia y coriorretinitis. En
1) La anemia fisiológica es más intensa y más precoz en los RNT que la TC craneal se observan calcificaciones periventriculares sin
en los RNPT. otros hallazgos. Con respecto al cuadro que parece presentar
2) El cuadro de anemia fisiológica neonatal está en relación con la este paciente, señale la verdadera:
hemólisis de glóbulos rojos y con un déficit de eritropoyetina.
3) En los RNPT está indicada la administración de suplementos de 1) Es la segunda causa más frecuente de infección viral congénita.
hierro a partir de los 2 meses de vida. 2) El 90% de los recién nacidos infectados estarán asintomáticos al
4) La anemia fisiológica puede precisar transfusión en algunos nacer.
casos. 3) La manifestación más tardía y característica es una hipoacusia de
5) La betatalasemia no se manifestará antes de los 6 meses de transmisión.
vida. 4) La afectación ocular típica es una retinitis pigmentosa.
5) Estos niños suelen desarrollar macrocefalia evolutivamente.
21. Señale la afirmación FALSA, de entre las siguientes, respecto a
la isoinmunización Rh: 27. ¿Cuál de las siguientes manifestaciones NO forma parte de la
tétrada de Sabin?:
1) La enfermedad hemolítica grave suele afectar al segundo hijo.
2) Suele cursar con ictericia, que aparece en las primeras 24 horas 1) Coriorretinitis.
de vida. 2) Cataratas.
3) Debe hacerse profilaxis con gammaglobulina específica anti-D 3) Crisis convulsivas.
antes de que pasen 72 horas del parto, cuando el test de Coombs 4) Hidrocefalia.
directo es positivo. 5) Calcificaciones cerebrales.
4) Cuando las cifras de bilirrubina superan los 20 mg/dl, es necesario
realizar una exanguinotransfusión. 28. Un RN de 38 semanas de edad gestacional y 1.750 g, presenta
5) El 90% de los casos de incompatibilidad se deben al antígeno D. erupción púrpuro-petequial, hepato-esplenomegalia y se aus-
culta un soplo continuo áspero en 2º espacio intercostal izquierdo.
22. Respecto a la hipoglucemia del hijo de madre diabética, señale En la Rx de tórax se objetiva cardiomegalia con aumento de la
cuál de las siguientes afirmaciones es FALSA: vascularización pulmonar así como imágenes lineales radiotrans-
parentes con aumento de densidad ósea en la zona metafisaria
1) Se observa en el 75% de los hijos de diabéticas y en el 25% de los del húmero sin reacción perióstica. En relación con el cuadro que
hijos de madres con diabetes gestacional. presenta este paciente, señale la respuesta FALSA:
2) Aparece en las primeras 24 horas de vida y suele ser asintomá-
tica. 1) Cuando la madre contrae la infección en el tercer trimestre aumenta
3) Se piensa que es debida a hiperinsulinismo fetal. el riesgo de infección del feto, pero el cuadro clínico es más leve.

CTO Medicina • C/Nuñez de Balboa 115 • 28006 Madrid • Tfno. (0034) 91 782 43 30/33/34 • E-mail: secretaria@ctomedicina.com • www. ctomedicina.com 3

Willy Aguilar
Test 1ª vuelta distancia
Pediatría
2) Si la infección se produce después de la 12ª semana de gestación,
es poco probable la asociación de una cardiopatía congénita. 1) El niño aproximadamente duplica su peso al 5º mes y lo triplica al
3) La presencia de IgM específica anti-rubéola refleja la producción año de vida.
en útero de anticuerpos por el feto. 2) El peso del RN puede disminuir un 5-10% en la primera semana
4) El lactante puede permanecer con infección crónica durante meses de vida por pérdida de líquido extravascular.
después del nacimiento. 3) Los reflejos precoces o arcaicos presentes al nacimiento suelen
5) El virus puede aislarse en sangre. desaparecer entre los 3 y 4 meses de edad.
4) Hacia los 2 meses suele haber hecho su aparición la sonrisa
29. De los siguientes, el factor más relacionado con el crecimiento social.
intrauterino retardado tipo simétrico es la: 5) Duplican la talla del nacimiento poco después de cumplir el primer
año de edad.
1) Infección congénita.
2) Isoinmunización Rh. 35. Con respecto al calostro, es FALSO que:
3) Hipertensión.
4) Diabetes con vasculopatía. 1) Se denomina así a la secreción mamaria durante los primeros 2-4
5) Edad de la mujer por encima de 35 años. días postparto.
2) Contiene más proteínas y más carbohidratos que la leche humana
madura.
30. Un RN hijo de madre prostituta presenta en el 7º día de vida un
3) Su densidad es mayor que la de la leche humana madura.
cuadro de fiebre con letargia y rechazo de las tomas así como
4) Las características de leche humana madura se adquieren alrededor
convulsiones. A la exploración destaca hepatoesplenomegalia
de la 3ª-4ª semana.
con fontanela anterior abombada y lesiones vesiculares en calota
5) Posee factores inmunológicos.
craneal. ¿Cuál sería la causa más probable de este cuadro?:

1) Infección por VIH. 36. Respecto a las diferencias entre la leche materna y la leche de
2) Infección por CMV. vaca, señale la afirmación correcta:
3) Infección por VHS tipo I.
4) Infección por VHS tipo II. 1) La leche de vaca tiene un contenido energético 2 veces superior
5) Exantema tóxico del RN. al de la leche materna.
2) La leche de vaca contiene una cantidad de calcio superior a la
31. Un recién nacido de tres semanas de vida que desprendió leche materna.
el cordón umbilical a los dieciséis días y cuyo ombligo no ha 3) La leche materna contiene mayor cantidad de vitamina K que la
cicatrizado, presenta una pequeña tumoración sonrosada con leche de vaca.
secreción seromucosa con un pH de 7,4. ¿Cuál de los siguientes 4) La leche de vaca es más rica en hidratos de carbono que la leche
diagnósticos le parece más probable?: materna.
5) La leche materna tiene una relación caseína/seroproteínas de
1) Onfalocele. 70/30.
2) Granuloma umbilical.
3) Quiste o persistencia del uraco. 37. La estatura de un niño de 4 años está por debajo del P3. Su
4) Gastrosquisis. velocidad de crecimiento es normal. La edad ósea es de 2 años. El
5) Ombligo amniótico. padre presentó un retraso en la pubertad, aunque actualmente
su talla está dentro de los límites normales. Este proceso se
32. Recién nacida, de 3 semanas de vida, que presenta valores de TSH denomina:
de 75 mcgr/ml ( 0.3 –5 ) en el screening neonatal de hipotiroidismo.
Con respecto a esta situación, es FALSO: 1) Hipopituitarismo primario.
2) Hipopituitarismo secundario.
1) Su causa más frecuente es la disgenesia tiroidea. 3) Retraso constitucional del crecimiento.
2) Situaciones de dismorfogénesis pueden asociarse a sordera neu- 4) Talla baja genética.
rosensorial. 5) Síndrome de Laron.
3) Un tratamiento precoz supone un mejor pronóstico neuroló-
gico. 38. Acude a su consulta un padre con su hijo de 8 años preocupado
4) Los signos clínicos del hipotiroidismo están ya presentes inme- porque dice que su hijo es el niño más bajo de toda su clase.
diatamente al nacimiento. Su talla actual se encuentra en el percentil 2 y su talla genética
5) En situaciones de mal control pueden acabar desencadenando está situada en el percentil 3. Al realizar el estudio de la edad
una pubertad precoz. ósea del niño se ve que ésta coincide con su edad cronológica.
¿Qué cree usted que explicaría la talla baja de este niño?:
33. En relación con el desarrollo psicomotor en el niño, señale la
respuesta FALSA: 1) Un hipocrecimiento por deficiencia de la hormona de creci-
miento.
1) La sonrisa referencial aparece ente el 1º y 2º mes. 2) Una alteración cromosómica.
2) El sostén cefálico se inicia a los tres meses. 3) Un hipotiroidismo congénito.
3) La sedestación estable se consigue a los 6-8 meses. 4) Un retraso constitucional del crecimiento.
4) Emite sus primeros fonemas con 3-4 semanas. 5) Una talla baja familiar.
5) Realiza frases cortas con 2 años.
39. Niña de 2 años acude a urgencias por cuadro de disnea, estridor
34. En relación con el crecimiento y el desarrollo del niño en su primer y tos perruna. En la exploración destaca rinorrea intensa, amíg-
año de vida, ¿cuál de las siguientes afirmaciones es falsa?: dalas hipertróficas con exudado puntiforme. Fiebre de 38,5º. A

CTO Medicina • C/Nuñez de Balboa 115 • 28006 Madrid • Tfno. (0034) 91 782 43 30/33/34 • E-mail: secretaria@ctomedicina.com • www. ctomedicina.com 4

Willy Aguilar
Test 1ª vuelta distancia
Pediatría
la auscultación pulmonar se aprecia hipoventilación bilateral y 1) Enfermedad celíaca.
estridor inspiratorio. El diagnóstico más probable será: 2) Tuberculosis.
3) Asma.
1) Traqueítis bacteriana. 4) Fibrosis quística.
2) Laringitis supraglótica. 5) Déficit de alfa 1 antitripsina.
3) Laringitis aguda.
4) Laringitis estridulosa. 44. Sobre la traqueítis bacteriana, todo es cierto, EXCEPTO:
5) Epiglotitis.
1) La causa más frecuente es el Haemophilus influenzae.
40. Ante un niño de 2 años de edad que, estando previamente bien, 2) Debe sospecharse ante todo crup viral con fiebre que no responde
se despierta bruscamente por la noche con tos perruna, estridor y al tratamiento.
dificultad respiratoria, pulso acelerado y afebril, pensaremos en: 3) No estarán presentes los hallazgos clínicos de una epiglotitis.
4) Será necesario instaurar tratamiento antibiótico.
1) Laringotraqueítis aguda. 5) En ocasiones puede requerir uso de vía aérea artificial.
2) Laringitis espasmódica.
3) Epiglotitis aguda. 45. Señale la respuesta FALSA, al hablar de la fibrosis quística:
4) Difteria.
5) Traqueítis bacteriana. 1) La lesión anatomopatológica pulmonar más precoz es la bron-
quiolitis.
41. Varón de 5 años de edad que, estando previamente bien, des- 2) Un 10% de los casos debutan con un íleo meconial.
pierta por la noche con fiebre de 39,8ºC, voz apagada, intenso 3) La función endocrina del páncreas se altera con el tiempo.
babeo, estridor inspiratorio con escasa tos. A la exploración 4) La aspergilosis broncopulmonar alérgica es excepcional en estos
destaca mal estado general, estridor inspiratorio e intenso tiraje pacientes y se trata con antifúngicos como el voriconazol.
supraesternal y subcostal, así como tendencia a la hiperextensión 5) Pueden presentar deshidratación con alcalosis hipoclorémica en
del cuello. Se realiza radiografía lateral de faringe, observando ambientes calurosos.
ocupación de espacio supraglótico. En el hemograma destaca
una leucocitosis con desviación izquierda. ¿Qué medida tera- 46. En el tratamiento de la fibrosis quística, NO es cierto que:
péutica NO estaría aconsejada en este caso?:
1) La fisioterapia respiratoria alarga la supervivencia.
1) Aplicar oxígeno e intentar obtener una vía aérea artificial en qui- 2) Los productos enzimáticos micronizados disminuyen la esteatorrea.
rófano. 3) Se necesita aportar suplementos de vitaminas liposolubles a la
2) Iniciar tratamiento antibiótico con ceftriaxona. dieta.
3) Aplicar adrenalina racémica y corticoides. 4) La colonización de la vía respiratoria por P. aeruginosa es fácilmente
4) Usar las medidas generales para descender la hipertermia que revertida con la antibioterapia adecuada.
sufre el paciente. 5) En ocasiones son necesarios lavados y aspiración traqueobronquial
5) Mantener tranquilo al paciente. broncoscópica.

42. Lactante varón de 3 meses de edad que desde hace dos días 47. Varón de 13 meses que desde hace 2 días presenta fiebre de
presenta cuadro de rinorrea serosa con estornudos y tos seca 38º C y síntomas catarrales. Hoy acude por presentar vómitos
acompañado de rechazo de las tomas. Hoy acude a Urgencias no biliosos y 6 deposiciones en las últimas 2 horas, líquidas, no
por cuadro de dificultad respiratoria con taquipnea y tos sibilante fétidas, sin moco ni pus ni sangre. Respecto al cuadro que pre-
paroxística. Exploración física: 37,7ºC, regular estado general, senta este paciente, señale cuál de las siguientes afirmaciones
acianótico, con aleteo nasal y tiraje sub e intercostal. Ausculta- es FALSA:
ción pulmonar: sibilancias espiratorias difusas con espiración
alargada. Se realiza hemograma con 3,5 millones de hematíes; 1) El agente causal más frecuente en la infancia es el rotavirus.
Hb de 10 g/dl; Hto de 32%; 7.000 leucocitos/mm3 con 50% 2) Suele ser un proceso autolimitado, que cede en 3-10 días.
neutrófilos, 35% linfocitos y 5% monocitos. En la Rx de tórax 3) Puede causar deshidratación isotónica con acidosis.
destaca hiperinsuflación pulmonar. Señale la FALSA respecto 4) Es frecuente que tienda a la cronificación.
a la evolución clínica de esta enfermedad: 5) El diagnóstico puede hacerse mediante detección rápida de antí-
geno de rotavirus en heces.
1) La fase más crítica son las primeras 48-72 horas desde el comienzo
de la tos y la disnea. 48. Lactante de 3 meses de edad, alimentado al pecho hasta hace
2) En menores de un mes puede cursar como apnea con escasez de 1 semana en que, debido a una hipogalactia, se decide suple-
clínica respiratoria. mentar con una fórmula artificial. Hoy acude a Urgencias por
3) Después de la fase crítica mejoran muy rápidamente. presentar anorexia, vómitos, pérdida de peso así como exantema
4) La ribavirina podría usarse en lactantes con cardiopatías congénitas urticarial en tronco. Entre sus antecedentes destaca RNT, nacido
o displasia broncopulmonar. por cesárea y haber iniciado alimentación oral a las 4 horas de
5) Las complicaciones bacterianas como la bronconeumonía u OMA vida con fórmula artificial. Con respecto a la patología actual,
son frecuentes. es FALSO que:

43. Paciente de 2 años de edad consulta por estancamiento ponde- 1) No suele haber antecedentes familiares de atopia.
roestatural (<P3). Entre los antecedentes personales destacan en 2) La proteína comúnmente implicada es la betalactoglobulina.
el último año episodios repetidos de bronquiolitis con regular 3) Las pruebas cutáneas serán positivas.
recuperación posterior, persistiendo una tos crónica. Además 4) El diagnóstico definitivo se realiza mediante un test de supresión-
presenta diarrea intermitente sin acompañarse de fiebre ni provocación.
vómitos. ¿Cuál sería el diagnóstico más probable?: 5) Suele ser un proceso transitorio.

CTO Medicina • C/Nuñez de Balboa 115 • 28006 Madrid • Tfno. (0034) 91 782 43 30/33/34 • E-mail: secretaria@ctomedicina.com • www. ctomedicina.com 5

Willy Aguilar
Test 1ª vuelta distancia
Pediatría
deposición semilíquida sanguinolenta. Exploración física: afebril;
palpación de una masa alargada y mal definida, localizada en
49. RN de pocas horas de vida que presenta exceso de secreciones hipocondrio derecho. En la Rx de abdomen aparece silencio
orales, episodios de cianosis y atragantamiento durante las aéreo en hipocondrio derecho. Señale cuál de las siguientes
tomas junto con un abdomen muy distendido y timpánico a afirmaciones es cierta respecto al cuadro que presenta este
la percusión. Entre sus antecedentes destacan polihidramnios paciente:
materno durante la gestación e imposibilidad de pasarle una
sonda nasogástrica hasta el estómago. En relación con la pato- 1) Debe retrasarse el tratamiento durante 4-6 horas debido a la alta
logía que sospecha, refiera cuál de las complicaciones siguientes incidencia de reducción espontánea.
al tratamiento quirúrgico es la más frecuente: 2) La recidiva es menos frecuente después de la reducción hidrostática
que tras la reducción quirúrgica.
1) Fístula anastomótica. 3) Si hay signos de obstrucción intestinal de 48 horas de evolución,
2) Estenosis esofágica. es preferible la reducción quirúrgica.
3) Fístula traqueoesofágica recidivante. 4) Una invaginación asociada a divertículo de Meckel suele reducirse
4) Reflujo gastroesofágico. satisfactoriamente con enema opaco.
5) Traqueomalacia. 5) La mortalidad es muy elevada, aunque la reducción tenga lugar
en las primeras 24 horas.
50. Varón de 3 meses que consulta por vómitos desde hace 1 mes.
Vomita después de todas las tomas, sin fuerza; come siempre 55. Niño de 4 meses es llevado a consulta por estreñimiento crónico
con buen apetito y presenta adecuado desarrollo ponderoes- desde el nacimiento. Entre sus antecedentes personales destacan
tatural, sin pérdida de peso. Realiza dos deposiciones diarias tapón meconial al nacimiento que se resolvió con enemas.
con heces de consistencia, aspecto y volumen normal. Entre sus Alimentación materna exclusiva. Talla y peso <P3. ¿Cuál de los
antecedentes destaca estar alimentado con fórmula artificial siguientes datos NO apoyaría el diagnostico?:
desde el mes de vida. Su actitud sería:
1) Biopsia rectal patológica.
1) Cambiar su fórmula por un hidrolizado de proteínas de vaca. 2) Heces acintadas.
2) Hacer endoscopia para descartar esofagitis. 3) Ampolla sin heces en el tacto rectal.
3) Solicitar ecografía abdominal y/o tránsito digestivo. 4) Relajación del esfínter anal interno.
4) Realizar manometría esofágica. 5) Permanencia del bario en colon durante más de 24 horas al realizar
5) Indicar a la madre medidas posturales y usar espesante de la un tránsito digestivo.
leche.
56. Paciente de 12 meses que, de forma progresiva, inicia un cuadro de
51. Señale la respuesta verdadera respecto a la estenosis hiper- inapetencia, mal carácter, retraso del crecimiento ponderoestatural,
trófica de píloro: así como diarrea abundante, fétida y de aspecto grasiento. Explora-
ción física: afebril, regular estado general, palidez cutaneomucosa,
1) El signo radiológico frecuente es el llamado “signo de la cuerda”. pérdida de masa muscular proximal y abdomen distendido. Señale
2) Es más frecuente en niñas. lo FALSO respecto a la enfermedad celíaca:
3) Presenta alcalosis metabólica hiperclorémica con hipopota-
semia. 1) Existe una predisposición genética, relacionada con los HLA B8,
4) La prueba diagnóstica de elección es la Rx de abdomen. DR7, DR3 y DQ2.
5) El tratamiento consiste en la pilorotomía submucosa de Rams- 2) Hay una mayor prevalencia de la enfermedad en niños con déficit
tedt. selectivo de IgA o diabetes mellitus.
3) El período más común de presentación es entre los 6 meses y los
52. RN con síndrome de Down presenta vómitos biliosos a las 36 2 años de edad.
horas de vida. A la exploración destaca abdomen excavado. El 4) Los anticuerpos antiendomisio Ig A tienen una alta especificidad
diagnóstico más probable es: y sensibilidad.
5) El trigo, arroz, cebada y centeno deben ser excluidos de la dieta
1) Estenosis hipertrófica de píloro. para toda la vida.
2) Enfermedad de Hirschprung.
3) Divertículo de Meckel. 57. Ante una enferma con clínica sospechosa de enfermedad celíaca,
4) Atresia duodenal. señale lo correcto respecto a su diagnóstico:
5) Invaginación intestinal.
1) Ante la sospecha muy alta con clínica florida, se debe excluir el
53. Con respecto al divertículo de Meckel, es FALSO que: gluten de la dieta, sin necesidad de realizar biopsia.
2) Los Acs antitransglutaminasa son los más específicos.
1) Afecta al 1-2% de la población. 3) La anatomía patológica de la muestra de biopsia es patognómo-
2) Se localiza en íleon a unos 75 cm de la válvula ileocecal. nica.
3) Se denomina hernia de Littré cuando el divertículo de Meckel se 4) Antes de realizar la primera biopsia, es necesario excluir el
aloja en una hernia inguinal indirecta. gluten.
4) Su manifestación clínica más frecuente es una hemorragia gas- 5) El control de la enfermedad se basa en la realización de biopsias
trointestinal acompañada de dolor abdominal. anuales.
5) El método diagnóstico más útil es la gammagrafía con Tc99.
58. Neonato varón, de tres semanas de vida, acude a su consulta
54. Un varón de 14 meses, previamente sano, de forma brusca por ictericia con coluria e hipocolia. A la exploración se palpa
presenta crisis de llanto con encogimiento de piernas y palidez, hepatomegalia. En la gammagrafia con HIDA, la captación es
separados por períodos asintomáticos de duración variable en normal, pero la excreción es nula. Sobre la entidad que sospecha,
los que permanece decaído. Ha presentado dos vómitos y una señale la opción FALSA:

CTO Medicina • C/Nuñez de Balboa 115 • 28006 Madrid • Tfno. (0034) 91 782 43 30/33/34 • E-mail: secretaria@ctomedicina.com • www. ctomedicina.com 6

Willy Aguilar
Test 1ª vuelta distancia
Pediatría

5) Si el reflujo es masivo, puede llevar a IRC en la infancia y puede


1) Se puede asociar a poliesplenia. requerir cirugía.
2) El tratamiento definitivo es la hepatoportoenterostomía.
3) En la biopsia es posible hallar células gigantes.
4) Es infrecuente la aparición de varios casos en una misma familia.
63. Varón de 2 años en cuya historia clínica destaca haber pade-
cido un proceso diarreico en los días previos. En la exploración
5) En el sondaje duodenal se suele observar ausencia de bilis.
destaca palidez, estupor y hematuria. Ha presentado además
una convulsión focal. Respecto al cuadro que usted sospecha,
59. Paciente de 5 meses que acude por presentar, desde hace 2 señale cuál de las siguientes opciones es FALSA:
días, hasta 10 deposiciones al día, líquidas sin moco ni sangre,
así como dolor medio abdominal sin relación temporal.
1) Es la causa más frecuente de IRA en los niños pequeños.
Entre sus antecedentes destaca diarrea sanguinolenta hace
2) Existe anemia hemolítica microangiopática y trombopenia dentro
1 semana. En la exploración destaca: afebril, buen estado
del cuadro clínico.
general, distensión abdominal con abundantes ruidos
3) La trombosis bilateral de las venas renales puede provocar un
hidroaéreos y eritema en región del pañal. En el examen
cuadro similar.
macroscópico de heces se detecta presencia de azúcares
4) El empleo de corticoides supone el tratamiento de elección.
reductores. Este cuadro es:
5) La aparición de recidivas es muy poco frecuente.
1) Déficit de sacarasa-isomaltasa.
64. Varón de 5 años presenta de forma súbita dolor en teste derecho,
2) Déficit aislado de isomaltasa.
sin antecedente de traumatismo previo. Exploración: afebril,
3) Déficit de lactasa.
teste tumefacto, doloroso a la exploración, con ausencia del
4) Déficit de enteroquinasa. reflejo cremastérico. Señale la afirmación FALSA:
5) Gastroenteritis por rotavirus.
1) En el Eco-Doppler se observaría disminución del flujo sanguíneo
60. Lactante de 1 mes de edad, nacido tras un embarazo controlado en ese testículo.
y normal, a las 35 semanas de edad gestacional, que acude a 2) El tratamiento consiste en antibióticos, antiinflamatorios, y tras la
su consulta para la “Revisión del niño sano” correspondiente. fase aguda, cirugía.
Todo parece estar bien salvo que los testículos se encuentran 3) La mayor incidencia se produce en la niñez tardía y adolescencia
en el canal inguinal, y aunque sí lo hacen con tracción manual, temprana.
espontámente no bajan al escroto. ¿Cuál es la actitud más ade- 4) En mayores de 13 años, se debe hacer diagnóstico diferencial con
cuada en este momento?: una epididimitis.
5) En el periodo neonatal, generalmente tiene mal pronóstico.
1) Si la alteración es bilateral, hay que consultar con el cirujano
cuanto antes, para que realice una orquidopexia, por el riesgo de 65. Un niño de 2 años de edad es traído a Urgencias por petequias
esterilidad y degeneración maligna. generalizadas sin otros signos de diàtesis hemorrágica. Refieren
2) Hay que esperar, porque es posible que espontánemente se catarro de vías altas y fiebre 2 semanas antes pero en la actualidad
complete el descenso testicular. de encuentra afebril, con buen estado general, y el resto de la
3) Trataría con HCG vía intramuscular a días alternos, y si no hay exploración es normal. Un hemograma muestra hemoglobina
respuesta, mandaría al cirujano para la extirpación de ambos 14 mg/dL, leucocitos 9400/mm3 y plaquetas 34.000 mm3. Los
testes. tiempos de protrombina y de troboplastina parcial activada
4) Se trata de unos testículos en ascensor, lo cual no requiere ningún son normales. Ante este cuadro clínico, una de las siguientes
tipo de tratamiento ni seguimiento. afirmaciones es cierta:
5) Trataría con testosterona y HCG, y si no hay respuesta, consultaría
con los cirujanos para su descenso. 1) El diagnóstico más probable es el de púrpura trombótica trom-
bocitopénica.
61. Lactante de 3 meses consulta por cuadro de vómitos y febrícula 2) Requiere de transfusión urgente de concentración de plaquetas.
de 48 horas de evolución, con progresiva pérdida del apetito. 3) La evolución más probable es a la recuperación espontánea.
En la exploración no se objetiva foco infeccioso. Hemograma 4) Debe practicarse un TAC craneal para descartar hemorragia
con leucocitosis y desviación a la izquierda. Sistemático de intracraneal.
orina: leucocitos y nitritos positivos. ¿Cuál sería la actitud más 5) El pronóstico depende de la precocidad del tratamiento.
adecuada?:
66. Los tumores malignos son la principal causa de muerte entre el
1) Tratamiento con antitérmicos y ver evolución. año y los 15 años de edad en los países desarrollados. Señale la
2) Realizar punción lumbar. afirmación INCORRECTA respecto a su epidemiología:
3) Diagnóstico de infección de orina y administrar antibióticos v.o.
4) Realizar punción suprapúbica e iniciar antibióticos. 1) La leucemia linfoblástica aguda es la neoplasia maligna más
5) Descartar reflujo gastroesofágico. frecuente en la infancia, y muestra un ligero predominio en
varones.
62. Es FALSO respecto al reflujo vesicoureteral en el niño: 2) La incidencia más alta de enfermedad de Hodgkin se da en ado-
lescentes y adultos jóvenes.
1) Puede sospecharse intraútero al ver dilatación pielocalicial en la 3) El neuroblastoma es el tumor sólido más frecuente en la infancia
ecografía obstétrica. fuera del sistema nervioso central.
2) Suele dar lugar a infecciones urinarias de repetición. 4) El sarcoma de Ewing es el tumor óseo primario maligno más
3) La indicación quirúrgica parte del número anual de recidivas de frecuente a partir de los 10 años, seguido del osteosarcoma.
infección urinaria. 5) El rabdomiosarcoma es el sarcoma de tejidos blandos más fre-
4) Puede ser útil hacer profilaxis antibiótica si se han repetido varias cuente en la infancia, siendo la cabeza y el cuello las principales
infecciones urinarias. localizaciones.

CTO Medicina • C/Nuñez de Balboa 115 • 28006 Madrid • Tfno. (0034) 91 782 43 30/33/34 • E-mail: secretaria@ctomedicina.com • www. ctomedicina.com 7

Willy Aguilar
Test 1ª vuelta distancia
Pediatría

67. Niña de 2 años presenta movimientos mioclónicos de extremi- 72. Niña de 5 años presenta cuadro catarral con fiebre de 37,5ºC.
dades y sacudidas desordenadas de los ojos. En la exploración A la exploración presenta adenopatías retroauriculares y cervi-
se palpa masa abdominal en línea media y flanco derecho. Se le cales dolorosas a la palpación e hiperemia conjuntival. 48 horas
realiza una TC abdominal, evidenciándose una masa de consis- después, aparece un exantema morbiliforme en cara. ¿Cuál
tencia mixta y calcificaciones en su interior. Señale la afirmación de las siguientes afirmaciones sobre la epidemiología de esta
falsa: enfermedad es cierta?:

1) Es el tumor extracraneal más frecuente en la infancia. 1) Las personas con enfermedad subclínica no son contagiosas.
2) La edad más frecuente al diagnóstico son los 2 años. 2) Los niños se afectan con mayor frecuencia que las niñas.
3) La presentación con un síndrome de mioclonus- opsoclonus 3) Los anticuerpos maternos protegen los 6 primeros meses de
ensombrece el pronóstico. vida.
4) Las catecolaminas en orina estarán elevadas. 4) Un solo ataque no suele conferir inmunidad de duración perma-
5) Se asocia con deleción del cromosoma 1 y anomalías del 17. nente.
5) La relación de enfermedad no aparente/manifiesta es de uno a
68. Señale la opción FALSA respecto al tumor de Wilms: uno.

1) Se asocia a deleción del cromosoma 11, ya sea en las células 73. Niño de 11 años, que consulta por fiebre, dolor de garganta
tumorales o en todas las células del organismo. y cansancio, de 5 días de evolución. En la exploración física,
2) Se asocia a aniridia y anomalías genitourinarias. destacan unas amígdalas hipertróficas con exudado en sábana,
3) La HTA es un hallazgo frecuente. adenopatías dolorosas axilares, inguinales, y las más grandes,
4) El diagnóstico definitivo nos lo da la biopsia. las cervicales, y hepatoesplenomegalia de 3-4 cm por debajo
5) Suele tratarse de una masa abdominal palpable asintomática, con de los rebordes costales. Pide un análisis de sangre, y lo pri-
o sin metástasis pulmonares. mero que le mandan del laboratorio es el hemograma: Hb 8,1
g/dl; Hto 27%; leucocitos totales 1800/ mm3 (600 neutrófilos);
69. Un niño de 5 años, previamente sano, debuta bruscamente con plaquetas 71000/ mm3. De momento no dispone de frotis de
palidez, astenia y anorexia; se le practica un hemograma y se sangre periférica, ni de reactantes de fase aguda, pero ya tiene
evidencia anemia, leucopenia y trombopenia. En la exploración una sospecha diagnóstica. Señale cuál es:
se aprecia hepatoesplenomegalia. ¿Qué diagnóstico le parece
más probable?: 1) Mononucleosis infecciosa.
2) Leucemia linfoblástica aguda.
1) Anemia ferropénica. 3) Linfoma en fase leucémica.
2) Linfoma no Hodgkin. 4) Hemoglobinuria paroxística nocturna (HPN).
3) Aplasia medular adquirida. 5) Aplasia medular por antibióticos.
4) Aplasia medular congénita.
5) Leucemia aguda. 74. Un niño de 4 años acude al pediatra por aparición de lesiones
papulosas rojas, algunas con vesículas blanquecinas no umbilicadas
70. Con respecto al tumor de Wilms, es FALSO que: en tronco y mucosa oral, muy pruriginosas. Dos días antes presen-
taba cuadro catarral con fiebre moderada. Respecto al cuadro que
1) Puede cursar con poliglobulia. presenta este niño, todo lo siguiente es cierto, EXCEPTO:
2) En la UIV se observa una masa dependiente del tejido renal.
3) La cirugía se indica, aun en presencia de metástasis pulmonares. 1) Está causado por un poxvirus, el virus de la varicela zoster.
4) Posee gran tendencia a presentar calcificaciones intraneoplá- 2) Las lesiones costrosas no contienen virus viables.
sicas. 3) La complicación más frecuente es la sobreinfección de las lesiones
5) Los tumores bilaterales son más frecuentes en los casos fami- cutáneas.
liares. 4) La neumonía varicelosa es poco frecuente en la infancia.
5) La encefalitis postvaricela que se presenta con signos cerebelosos
71. Niña de 6 años que presenta desde hace 3 días fiebre de 38º C tiene mejor pronóstico que si lo hace con signos cerebrales.
que no cede con antitérmicos, tos seca y facies congestiva. Hoy
acude por aparecer exantema maculopapuloso, rojo intenso, 75. Niño de 7 años presenta rash eritematoso en ambas mejillas,
no puntiforme, confluente, que no se blanquea a la presión tronco y zona proximal de extremidades, con tendencia a acla-
en la parte superior del tórax, cara y parte proximal de EESS. rarse en la zona central. Con respecto a esta enfermedad, señale
A la exploración. Tª 40,3ºC, adenopatías bilaterales en ángulo la afirmación FALSA:
mandibular y lesiones blanquecinas sobre base eritematosa en
mucosa subyugal. Respecto a la enfermedad que padece esta 1) Es producida por el parvovirus B19.
paciente, señale la afirmación FALSA: 2) El período de incubación es de 7 días.
3) El período prodrómico se caracteriza por fiebre alta.
1) El exantema comienza por la cara, tiene evolución descendente, 4) El exantema característico aparece en tres etapas.
y desaparece en el mismo orden en el que apareció. 5) Como complicación puede producirse una crisis aplásica grave.
2) La neumonía de células gigantes de Hecht es menos frecuente
que la neumonía por sobreinfección bacteriana. 76. Un lactante de 7 meses presenta súbitamente fiebre de 40ºC. No
3) La gravedad de la enfermedad está directamente relacionada con existe ningún otro síntoma y la exploración física es normal, salvo el
la intensidad y confluencia del exantema. hallazgo de una coriza ligera y adenopatías cervicales posteriores.
4) El prurito suele ser intenso. El niño parece encontrarse bastante bien. ¿Cuál de las siguientes
5) Entre las posibles complicaciones se encuentra la anergia cutánea afirmaciones describe mejor los factores diagnósticos que deben
y la reactivación de una TBC preexistente. considerarse en este caso?:

CTO Medicina • C/Nuñez de Balboa 115 • 28006 Madrid • Tfno. (0034) 91 782 43 30/33/34 • E-mail: secretaria@ctomedicina.com • www. ctomedicina.com 8

Willy Aguilar
Test 1ª vuelta distancia
Pediatría
4) El recuento de leucocitos suele elevarse en forma considerable
1) Si apareciera una erupción cutánea 24 horas después de comenzar con predominio de polimorfonucleares.
la fiebre, sería probable el diagnóstico de roséola o exantema 5) El período de incubación oscila entre 5 y 10 días.
súbito.
2) En la roséola infantil, el diagnóstico diferencial puede ser difícil,
puesto que en las primeras 36 horas cursa con leucocitosis, con 81. Con respecto al SIDA en la infancia, señala la opción correcta:
predominio de neutrófilos.
3) La fiebre podría corresponder a los pródromos de una rubéola. 1) El SIDA precoz es menos frecuente que el tardío y la clínica pre-
4) Si la fiebre dura 3 días, desaparece luego rápidamente y aparece una dominante es infecciosa.
erupción cutánea en ese momento, sería probable el diagnóstico 2) El SIDA tardío es más frecuente y predomina la clínica neuroló-
de eritema infeccioso. gica.
5) Es probable una escarlatina, pues es una enfermedad propia de 3) La neumonía por MAI es la infección oportunista más frecuente
niños de 6 a 18 meses. en el SIDA pediátrico.
4) La manifestación más frecuente en el SIDA infantil es la Hepatitis B.
77. Jennifer tiene 16 meses de edad y acude a Urgencias por pre- 5) La parotiditis es más frecuente en el SIDA infantil que en el del
sentar desde hace 2 días fiebre de 39ºC que su pediatra atri- adulto.
buyó a cuadro respiratorio. A la exploración presenta fiebre de
40ºC y contractura antiálgica en flexión de extremidad inferior 82. Sobre el diagnóstico de un niño VIH+, es FALSO que:
izquierda. Queda ingresada para estudio, y 4 días más tarde
persiste el cuadro febril, junto con edema indurado en ambos 1) Una IgG+ en un lactante de 10 meses indica infección activa por
pies, fisuras labiales, conjuntivitis no purulenta y exantema el VIH.
polimorfo en tronco. El diagnóstico más probable será: 2) La mayoría de los niños infectados presenta hipergammaglobu-
linemia policlonal precoz.
1) Rubéola. 3) La linfopenia e inversión del cociente CD4/CD8 son menos llama-
2) Mononucleosis infecciosa. tivas que en el adulto.
3) Púrpura de Schönlein-Henoch. 4) No es útil la determinación de IgA contra el VIH, porque no suele
4) Enfermedad de Kawasaki. aparecer hasta el 6º mes de vida.
5) Eritema infeccioso. 5) Los resultados positivos en la PCR han mostrado una correlación
positiva con el aislamiento del virus en cultivo.
78. Paciente varón de 5 años de edad que acude por presentar
cuadro febril y odinofagia con exudado blanquecino en ambas 83. Respecto a la vacuna triple vírica, señale la afirmación FALSA:
amígdalas que su pediatra trató con penicilina oral. Dos días des-
pués, presenta petequias y púrpura palpables, principalmente 1) Es una vacuna de microorganismos atenuados.
en miembros inferiores, así como artralgias en ambos tobillos 2) La alergia al huevo ha dejado de ser una contraindicación absoluta
que le impiden la marcha y dolor abdominal de tipo cólico. Los para su administración.
estudios complementarios revelan 13 g/dl de Hb; leucocitos 3) Está contraindicada su utilización en embarazadas y pacientes
10.500/mm3 con fórmula normal; plaquetas 485.000/mm3; VIH+.
tiempo de protrombina de 95% y tiempo parcial de trombo- 4) Su administración en el calendario actual se realiza a los 15 meses
plastina de 27 segundos (control 25 segundos). La causa más y a los 4 años.
probable de estos hallazgos es: 5) En zonas de elevada morbimortalidad puede administrarse la pri-
mera dosis a los 6 - 12 meses, con una segunda dosis de refuerzo
1) Reacción a la penicilina. después del año de vida.
2) Púrpura de Schönlein-Henoch.
3) Mononucleosis infecciosa. 84. Señale la afirmación FALSA respecto a la vacuna de la polio:
4) Enfermedad de Kawasaki.
5) Púrpura trombocitopénica idiopática. 1) La vacuna tipo Sabin es de virus vivos atenuados.
2) La vacuna oral produce inmunidad local (IgA) y general (IgG).
79. Respecto a la mononucleosis infecciosa, es FALSO que: 3) En el calendario actual se administra a los 2-4-6-18 meses y entre
los 4 y 6 años.
1) Hasta en el 80% hay aumento de transaminasas. 4) La vacuna tipo Salk está contraindicada en familiares de inmuno-
2) En la analítica aparece leucopenia. deprimidos.
3) Títulos elevados de anticuerpos frente al virus de Epstein-Barr. 5) En caso de diarrea o vómitos sería conveniente retrasar la vacu-
4) Existe un síndrome linfoproliferativo en varones que han sufrido nación con virus atenuados.
la infección por este virus.
5) La complicación más temida es la rotura esplénica. 85. Señale la afirmación cierta respecto a la vacuna DTP:

80. Un bebé de 3 meses, que recibe lactancia materna, presenta 1) La vacuna de la tos ferina está contraindicada en mayores de 18
una historia de accesos de tos con congestión facial, lagrimeo meses.
y acaba vomitando. Hace 2 semanas presentó cuadro catarral 2) No puede administrarse conjuntamente con otras vacunas.
de vías áreas altas. En la Rx tórax se aprecian breves infiltrados 3) La vacuna DTPa (acelular) tiene mayor riesgo de efectos secundarios
perihiliares. Con respecto a este cuadro, señale la respuesta que que la DTP, por lo que no se utiliza de forma rutinaria.
NO sea cierta: 4) A los 14 años es preciso vacunar con Td (tétanos y difteria adultos),
y no son necesarios nuevas dosis de recuerdo.
1) Puede aparecer al principio de la vida por falta de anticuerpos 5) No está contraindicada en pacientes inmuno-deprimidos.
maternos.
2) La fase catarral precede al período paroxístico. 86. Con respecto a la vacuna contra el Haemophilus influenzae tipo
3) La tos paroxística puede durar de 1 a 4 semanas. B, es FALSO que:

CTO Medicina • C/Nuñez de Balboa 115 • 28006 Madrid • Tfno. (0034) 91 782 43 30/33/34 • E-mail: secretaria@ctomedicina.com • www. ctomedicina.com 9

Willy Aguilar
Test 1ª vuelta distancia
Pediatría

1) Las nuevas vacunas conjugadas son aptas para la inmunización 1) Tetralogía de Fallot.
en lactantes mayores de 2 meses. 2) Coartación de aorta.
2) Es muy inmunógena. 3) Anomalía de Ebstein.
3) Sus efectos adversos son mínimos. 4) Trasposición simple de las grandes arterias.
4) Es necesario aplicarla después de los 5 años. 5) Atresia tricuspídea.
5) En el calendario vacunal debería administrarse a los 2, 4, 6 y 18
meses. 93. Lactante de 9 meses de edad que llega a Urgencias traído por
su madre, por somnolencia de 12 horas de evolución. No ha
87. Señale la afirmación falsa respecto a la vacuna del VHB: tenido fiebre, vómitos, ni ninguna otra sintomatología. En la
actitud de la madre destaca que parece darle poca importancia
1) La vacuna de VHB está realizada por ingeniería genética con efectos al estado del niño, contestando a las preguntas de la anam-
adversos mínimos. nesis con desgana, aunque espontáneamente expresa todas
2) La pauta de administración en caso de riesgo inmediato como las dificultades por las que está pasando para compaginar su
contacto con una jeringuilla sería 0, 1, 2 meses, y al año. papel de madre con su trabajo. En la exploración física, el niño
3) La vacuna VHB no está recomendada en mujeres embarazadas. está consciente y reacciona con irritabilidad a estímulos, pero
4) No está indicada su administración en niños mayores de 7 años. tiende a quedarse dormido. Tiene un hematoma periorbitario
5) Se puede administrar con la gammaglobulina anti VHB, siempre reciente, y varios pequeños antiguos en la región dorsolumbar,
que se administre en distinto punto. que la madre atañe a caídas accidentales, aunque el niño está
iniciando ahora la bipedestación con ayuda. ¿Qúe sospecharía
88. ¿Cuál sería la actitud más adecuada a seguir en los contactos inicialmente como causante del estado del niño?:
de un niño con meningitis meningocócica?:
1) Meningitis bacteriana.
1) Realizar quimioprofilaxis con ceftriaxona durante 2 días. 2) Malos tratos.
2) No existe quimioprofilaxis eficaz. 3) Enfermedad metabólica no diagnosticada descompensada.
3) Administración de la vacuna antimeningocócica en niños menores 4) Hipoglucemia.
de 5 años. 5) Hiponatremia.
4) Administración de rifampicina durante 2 días.
5) Administración de rifampicina durante 4 días. 94. Con respecto a la coartación de aorta, señale la FALSA:
89. ¿Cuál sería la actitud a seguir ante un niño de 4 años asintomático 1) El tipo más frecuente es la coartación de localización yuxtaductal.
con Mantoux negativo, hijo de un tuberculoso bacilífero?: 2) La mayor parte de los casos se muestran asintomáticos.
3) La tensión arterial en los miembros inferiores es mayor que en los
1) Observación y repetir Mantoux en 6 meses. superiores.
2) Realizar quimioprofilaxis primaria durante 2 meses con INH y 4) En ocasiones, la tensión arterial en el brazo derecho es mayor que
repetir el Mantoux posteriormente. en el izquierdo.
3) Realizar quimioprofilaxis primaria durante 6 meses con INH. 5) En la radiografía de tórax, con el tiempo, se visualizan escotaduras
4) Administrar tratamiento completo con tres fármacos durante 6 en los bordes inferiores de las costillas.
meses.
5) Vacunación con BCG.
95. Con respecto al desarrollo puberal normal, es verdadero:
90. Respecto al Síndrome de Muerte Súbita Infantil o del Lactante 1) El primer dato a la exploración sugestivo de inicio de la pubertad
(SMSL), señale cuál de los siguientes NO constituye un factor en las niñas es la pubarquia.
de riesgo: 2) El aumento del tamaño testicular en el varón es el primer signo
de pubertad.
1) Antecedentes familiares. 3) La pubertad suele iniciarse a los 8 años en las niñas y a los 9 en los
2) Prematuridad. niños.
3) Lactancia materna. 4) El vello sexual es el primer signo de pubertad.
4) El sexo masculino. 5) La ganancia de peso y talla es más acusada en las niñas que en los
5) El tabaquismo materno. niños.

91. La persistencia del ductus arterioso se asocia a todo lo siguiente, 96. Niña de 4 años que consulta por desarrollo mamario lentamente
EXCEPTO a: progresivo desde hace 2 meses, sin adrenarquia ni aumento de la
velocidad de crecimento. Los padres no lo relacionan con ningún
1) Pulsos periféricos saltones. desencadenante ni con otros signos-síntomas. La exploración
2) Mayor frecuencia en prematuros. física general es normal, con un estadio puberal II de Tanner (T2
3) Soplo continuo en el área pulmonar. P1 Aa). Se realiza una radiografía de mano y muñeca izquierdas,
4) Puede ser efectivo el tratamiento con indometacina. estimándose la edad ósea en 3,5-4 años. En la ecografía pélvica
5) Las prostaglandinas son el mejor tratamiento médico para con- se visualizan útero y ovarios de morfología y tamaño adecuados
seguir su cierre. a la edad de la paciente, sin quistes significativos. Los niveles
séricos de estradiol son de 40 pg/dl (N 10-30), y la FSH y la LH
92. Ante un recién nacido de dos días con cianosis, taquipnea sin basales, son normales. ¿Cuál es su diagnóstico de sospecha?:
soplo a la auscultación y que, en la radiografía de tórax, muestra
ligera cardiomegalia, con un pedículo cardíaco estrecho y 1) Pubertad precoz central, probablemente idiopática.
aumento del flujo sanguíneo pulmonar, el diagnóstico más 2) Pubertad precoz central, probablemente por hamartoma hipota-
probable es: lámico.

CTO Medicina • C/Nuñez de Balboa 115 • 28006 Madrid • Tfno. (0034) 91 782 43 30/33/34 • E-mail: secretaria@ctomedicina.com • www. ctomedicina.com 10

Willy Aguilar
Test 1ª vuelta distancia
Pediatría
3) Telarquia prematura aislada.
4) Pubertad precoz periférica.
5) Aunque no sepamos el índice de masa corporal (IMC) de la paciente,
lo más probable es que sea elevado y nos encontremos realmente
ante una adipomastia bilateral.

97. ¿Cuál de los siguientes cuadros no forma parte de diagnóstico


diferencial de talla baja en los niños?:

1) Síndrome de Turner.
2) Síndrome de Down
3) Síndrome de Silver-Russell.
4) Síndrome de Klinefelter.
5) Síndrome de Noonan.

98. Respecto a las convulsiones neonatales, señale la afirmación


FALSA:

1) Los RN con convulsiones suelen tener mal pronóstico.


2) El tratamiento consiste en medidas de sostén y fenobarbital o
diacepam.
3) Las crisis tónico-clónicas generalizadas son raras durante el primer
mes de vida.
4) La clínica es variable y el EEG puede ser la única forma para su
diagnóstico.
5) La causa más frecuente de crisis en el período neonatal es la
encefalopatía hipóxico-isquémica.

99. Acerca de la enuresis, señale cuál es la respuesta correcta:

1) Es más frecuente en el sexo femenino.


2) La enuresis secundaria es más frecuente entre los 5 y los 8 años
de edad, y tiene peor pronóstico que la primaria.
3) La predisposición genética es un factor de riesgo muy importante
para la enuresis nocturna.
4) La causa más frecuente de enuresis diurna es la infección de
orina.
5) El tratamiento de elección en la enuresis nocturna persistente es
la desmopresina administrada antes de acostarse.

100. Llega a Urgencias un niño de 18 meses con fiebre de 39º, que


ha presentado en su domicilio un episodio tónico-clónico gene-
ralizado de 3 minutos de duración, con posterior somnolencia.
Lo correcto a su llegada a Urgencias sería:

1) Administración de bolo de glucosa.


2) Administración de antitérmicos y diacepam.
3) Administración de fenitoína.
4) Realizar punción lumbar.
5) Administración de piridoxina.

CTO Medicina • C/Nuñez de Balboa 115 • 28006 Madrid • Tfno. (0034) 91 782 43 30/33/34 • E-mail: secretaria@ctomedicina.com • www. ctomedicina.com 11

Willy Aguilar
CTO Medicina • C/Nuñez de Balboa 115 • 28006 Madrid • Tfno. (0034) 91 782 43 30/33/34 • E-mail: secretaria@ctomedicina.com • www. ctomedicina.com 12

Willy Aguilar
Comentarios de Test a distancia 1ª vuelta
Pediatría

Comentarios de test a distancia 1ª vuelta

Pediatría
Pregunta 1.- R: 3
adultos, o incluso para los nacidos a término, sí lo son para los
El test de Apgar es un test consensuado para documentar el estado
pretérmino, por lo que es necesario conocer estas particularidades
del RN en momentos puntuales. Lo más frecuente es que en un
antes de administrar cualquier fármaco en estos pacientes.
RN sano se mida al minuto de vida y a los 5 minutos, pero si a los
5 minutos la puntuación es inferior a 7 se continúa midiendo a los
10-15 y 20 minutos. Se debe realizar a cualquier RN. Pregunta 4.- R: 3
Es importante conocer los parámetros que se puntúan y su valo- La respuesta 1 es cierta: es importante para el MIR tener claro las
ración. diferencias entre caput (edema de partes blandas, no limitado por
Este test no sirve para valorar el riesgo de mortalidad perinatal ni la suturas) y cefalohematoma (hematoma entre hueso y periostio,
probabilidad de daño neurológico. limitada por suturas).
La respuesta 2 es cierta: el traumatismo obstétrico más frecuente
Parámetros 0 1 2 es la fractura de clavícula. A la exploración lo más llamativo es la
Esfuerzo respiratorio Ausente Débil Llanto crepitación y el reflejo de Moro asimétrico.
FC Ausente <100 lmp >100 lmp La respuesta 3 es falsa: el llamado nódulo de Stromayer es un he-
Tono muscular Hipotonía Ligera flexión Movimientos activos matoma palpable a nivel del ECM. La mayoría de las veces se
Irritabilidad refleja No Muecas Tos
Color Palidez Acrocianosis Rosado reabsorbe espontáneamente, pero en ocasiones se puede fibrosar
acortando el músculo. El tratamiento se basa en la fisioterapia y
Pregunta 1. Test de Apgar.
muy rara vez requiere cirugía.
La respuesta 4 es cierta: la víscera que con mayor frecuencia se lesiona
Pregunta 2.- R: 4 es el hígado, pero la mayoría de las veces son pequeños hematomas
Si seguimos la tabla del comentario de la pregunta anterior obten- subcapsulares limitados que secundariamente pueden producir
dremos una puntuación de 4 en el test de Apgar realizado a este ictericia precoz prolongada, sin otra repercusión clínica. Recordar
recién nacido. para el MIR el cuadro clínico típico de la hemorragia suprarrenal:
niño grande, parto de nalgas, que en las primeras horas de vida
Pregunta 3.- R: 1 presenta clínica de hipotensión, sangrado y shock.
Los recién nacidos pretérmino tienen una mortalidad y una mor- Respuesta 5: las fracturas craneales lineales generalmente no precisan
bilidad mayor que los neonatos a término, en relación con su tratamiento quirúrgico, salvo que se traten de fracturas deprimidas
propia inmadurez, y con factores externos como por ejemplo las que produzcan lesiones en el parénquima.
infecciones, que pueden ser la causa de su prematuridad.
La alimentación de los prematuros debe realizarse de forma cautelosa. Pregunta 5.- R: 4
Siempre que no haya distrés respiratorio ni otras circunstancias Dentro de los fenómenos dermatológicos que podemos encontrar en
que contraindiquen su inicio, hay que procurar una alimenta- un neonato, y que no tienen significación patológica, se encuentran:
ción precoz para evitar la hipoglucemia, la deshidratación y la los quistes de milium, la mancha mongólica, los angiomas planos,
hiperbilirrubinemia; se puede intentar vía oral y continuar si hay el eritema tóxico y la melanosis pustulosa.
buena tolerancia, aunque en los menores de 32 semanas es muy El eritema tóxico es más propio de la raza blanca y consiste en vesi-
frecuente que no exista la coordinación adecuada y haya que culopústulas sobre base eritematosa que suelen respetar palmas y
colocar una SNG. plantas y tiende a desaparecer en la primera semana. La melanosis
La anemia fisiológica, aparece antes y con más intensidad que en
pustulosa es más frecuente en la raza negra, la erupción vesiculo-
los nacidos a término, por los menores depósitos de hierro y el pustulosa sí suele afectar a las palmas y plantas, y tiene tendencia
rápido crecimiento, por lo que a veces precisan ser tratados con a desaparecer en varias semanas.
eritropoyetina, que debe acompañarse siempre de suplementos
de hierro vía oral.
ERITEMA TÓXICO MELANOSIS PUSTULOSA
Las pérdidas insensibles de líquido están aumentadas respecto a los
neonatos nacidos a término, en relación inversa a la edad gesta- Aparece 1-3 días Nacimiento
cional, y sus necesidades diarias son mayores (de 70-100 ml/kg/ · Variable · Variable
Localización
día el primer día, aumentando hasta 150 ml/kg/día). · No palmoplantar · Sí palmoplantar
En todos los recién nacidos, pero más en los prematuros, los tóxicos Frotis Eosinófilos Neutrófilos
de eliminación renal, y en los prematuros también los de elimi-
Cultivo Estéril Estéril
nación hepática, deben darse en menor dosis o con intervalos
mayores. Algunos medicamentos que no son tóxicos para los Pregunta 5. Melanosis pustulosa Vs Eritema tóxico del RN

CTO Medicina • C/Nuñez de Balboa 115 • 28006 Madrid • Tfno. (0034) 91 782 43 30/33/34 • E-mail: secretaria@ctomedicina.com • www. ctomedicina.com 1

Willy Aguilar
Comentarios de Test a distancia 1ª vuelta
Pediatría
En los casos más graves se asocia con una alta mortalidad debido a
Pregunta 6.- R: 2 la hipertensión pulmonar que desarrollan.
El cuadro clínico se refiere a una taquipnea transitoria del RN, Sd de El tratamiento consiste en ventilación mecánica, antibióticos
Avery, pulmón húmedo o SDR tipo II. y medidas para disminuir la hipertensión pulmonar: alcali-
La presentación típica es un RN nacido por cesárea o por parto vagi- nizar, tolazolina y oxido nítrico inhalado. Algunos niños se
nal rápido, lo que supone que el tórax no ha estado sometido a la pueden beneficiar de la ECMO (oxigenación por membrana
presión positiva que exprime de líquido los alveolos del pulmón. La extracorpórea).
clínica se inicia en las primeras horas de vida, que cede y mejora con
medidas poco agresivas (oxígeno en incubadora). Suelen alcanzar Pregunta 10.- R: 1
la resolución clínica en un plazo máximo de 3 días. Caso clínico de una parálisis braquial superior, también conocida
Lo que establece el diagnóstico, aparte de la evolución clínica, es la Rx como parálisis de Erb-Duchenne. Se debe a la afectación de las
de tórax, donde se aprecia aumento de marcas vasculares, líquido raíces C5-C6 del plexo braquial. El recién nacido presenta el brazo
en las cisuras, SIN broncograma aéreo. Esto último es imprescindible en adducción y rotación interna con el reflejo de Moro ausente en
para distinguirlo de la Rx de la EMH (R1). el lado de la lesión (respuesta 1 correcta). Sin embargo, el reflejo
R3: El SAM es típico de RN postérmino, con antecedentes de estrés de prensión palmar suele estar presente al no verse afectadas las
en el momento del parto. raíces inferiores. En los casos en que se lesione también la raíz C4
R4: La PCF suele cursar con clínica marcada de cianosis que no res- puede asociarse con una parálisis frénica.
ponde a la administración de oxígeno. No hay una Rx de tórax
característica. PARÁLISIS BRAQUIAL P. ERB-DUCHENNE P. KLUMPKE
R5: el Sd de Wilson-Mikity es a efectos clínicos semejante al término
de DBP. Raíces (C4) - C5 - C6 C7 - C8 - (T1)
Pregunta 7.- R: 2 Brazo en adducción y
Clínica Mano caída
Estamos ante un cuadro típico de enfermedad de membranas hia- rotación interna
linas: RNPT con dificultad respiratoria intensa que aparece en las R. Moro No presente o asimétrico Presente
primeras horas de vida, que no mejora tras la administración de R. Prensión palmar Presente No presente
oxígeno. En la gasometría es llamativa la importante hipoxia. Para
Asociaciones C4 - Parálisis frénica T1 - S. Horner
el diagnóstico es necesario realizar una Rx de tórax, y en ocasiones,
la medición del cociente lecitina/ esfingomielina en secreciones Pregunta 10. Parálisis braquial
bronquiales, que debe ser inferior a 2.
No existe una imagen radiológica patognomónica. Lo más típico es Pregunta 11.- R: 5
encontrar un infiltrado retículo granular con broncograma aéreo. Estamos ante un RN con displasia broncopulmonar. La definición ya
Pero esto mismo nos lo podemos encontrar en una neumonía, aparece en el caso clínico: necesidad de oxígeno para mantener
por lo que a la hora de instaurar tratamiento debemos cubrir adecuadas saturaciones. Dentro de los factores de riesgo para este
esta posibilidad. cuadro figuran: la toxicidad del oxígeno en altas concentraciones,
La R3 corresponde al pulmón húmedo. inmadurez pulmonar y barotrauma.
La R4: El patrón de esponja en la Rx es típico de la DBP. El diagnóstico se establece por la historia clínica y la Rx de tórax, donde
La R5: corresponde a un neumotórax bilateral. es típico encontrar el llamado patrón de esponja (aparecen áreas
más claras que alternan con otras de mayor densidad).
Pregunta 8.- R: 4 El tratamiento se basa en el uso de diuréticos y broncodilatadores; el
El tratamiento de la EMH consiste en: uso de corticoides actualmente está muy debatido.
• Monitorización estrecha, control de líquidos. R5: la mayoría de los niños suelen tener un curso favorable, alcan-
• Intentar mantener una aceptable oxigenación (pO2 50-70 mmHg). zando la normalidad de la función pulmonar hacia los 2 años
Si no se consigue con oxígeno indirecto, será necesario intubar al de vida. Hay un pequeño porcentaje que evolucionan hacia un
paciente e iniciar ventilación mecánica. cuadro de hipertensión pulmonar persistente. Las dos causas
• Administración de surfactante intratraqueal en las primeras 24 más frecuentes de muerte en estos niños son la ICC derecha y la
horas de vida, pudiendo recibir hasta 4 dosis según evolución. bronquiolitis necrotizante.
Esta medida se ha visto que mejora la clínica de la EMH, pero que
no disminuye los casos de DBP. Pregunta 12.- R: 3
• Administración de antibióticos: ampicilina más gentamicina, Se trata de un caso típico de hernia diafragmática congénita (hernia
porque la clínica y la Rx pueden ser semejantes en el caso de una de Bochdalek). Consiste en un orificio grande en la parte poste-
sepsis con neumonía. rior izquierda del diafragma, a través del que pasan las vísceras
abdominales en el período fetal, dando lugar a una hipoplasia
La respuesta incorrecta es la 4. Las prostaglandinas en los neonatos pulmonar importante y un desplazamiento del mediastino al lado
están indicadas en caso de cardiopatías ductus dependientes, en contrario por compresión. Esto produce por una parte un gran
las que es IMPRESCINDIBLE mantener el ductus abierto. compromiso respiratorio, con disminución del número y tamaño
de unidades alveolares y sus arteriolas, que en cambio tienen
Pregunta 9.- R: 2 una capa muscular engrosada, dando lugar a una hipertensión
El caso clínico es el de un niño que ha sufrido un SAM. Lo típico es pulmonar intensa que ya está presente al nacimiento (respuesta
que se produzcan en RN postérmino, con un estrés en el momento 5). Además puede haber alteraciones hemodinámicas graves por
del parto, que estimula el peristaltismo intestinal y se produce la la compresión del mediastino.
eliminación de meconio intraútero. La hernia diafragmática congénita es más frecuente en algunos
Al nacimiento, el meconio se encuentra en las vías aéreas altas; si síndromes congénitos, como las trisomías 21, 13 y 18. En un
no se elimina a tiempo, este pasará al pulmón con las primeras 20-30% de los casos hay otras alteraciones asociadas (respuesta
respiraciones, produciendo un cuadro de alveolitis con riesgo de 1): neurológicas, cardiovasculares, y digestivas.
sobreinfección bacteriana, especialmente por E. coli, atelectasias El diagnóstico en la mayoría de los casos se hace por ecografía prenatal.
y atrapamiento aéreo con riesgo de neumotórax y neumome- Cuando no es así, el cuadro clínico típico es como el expuesto, y se
diastino. realiza una radiografía de tórax para confirmarlo. El neumotórax

CTO Medicina • C/Nuñez de Balboa 115 • 28006 Madrid • Tfno. (0034) 91 782 43 30/33/34 • E-mail: secretaria@ctomedicina.com • www. ctomedicina.com 2

Willy Aguilar
Comentarios de Test a distancia 1ª vuelta
Pediatría
es la patología que plantea el principal diagnóstico diferencial • A continuación se reflejan las causas más frecuentes de inctericia
tras la exploración física, aunque en este caso, el abdomen está según el momento de aparición:
distendido. Si tras la radiografía hay dudas con malformaciones 1as 24 horas Hemólisis.
pulmonares quísticas, hay que realizar una ecografía, y en algunos Infecciones: sepsis, TORCH.
casos un TAC. 2º-3er día Fisiológica.
El tratamiento debe comenzar con la estabilización respiratoria Infecciones: sepsis, TORCH.
y hemodinámica, intentando disminuir las resistencias vascu- Anemias hemolíticas.
lares pulmonares y el posible cortocircuito derecha-izquierda. 4º-7º día Sepsis.
Posteriormente se realiza la reparación quirúrgica, que antes se TORCH.
consideraba de urgencia. Obstrucción intestinal.
Las secuelas pulmonares y neurológicas son frecuentes. Práctica- Lactancia materna.
mente todos tienen reflujo gastroesofágico al nacer, y hasta un >1 mes Galactosemia, hipotiroidismo, lactancia materna,
20% necesitan una funduplicatura. metabolopatías, ictericia obstructiva, Gilbert,
Crigler-Najjar...
Pregunta 13.- R: 3 • Características de la ictericia fisiológica vs no fisiológica:
Con los datos que nos presentan en este caso clínico de RNPT con - Inicio en las primeras 24 horas: SIEMPRE patológico.
distress respiratorio y que de forma brusca desarrolla hipoten- - Predominio de bilirrubina directa: SIEMPRE patológico.
sión, bradicardia, cianosis, fontanela abombada y disminución • Otros datos de la no fisiológica:
del hematocrito, el diagnostico será de hemorragia de la matriz - Incremento mayor de 5 mg/24 horas.
germinal. - Bilirrubina en sangre de cordón mayor de 3 mg/dl.
La matriz germinal es una estructura transitoria, presente en el - RNT > 12 y RNPT > 14.
cerebro inmaduro en la cabeza del núcleo caudado, cerca de - Duración mayor de 14 días.
los ventrículos laterales) hasta la semana 34 de gestación.
Dicha matriz está constituida por vasos y es muy sensible a la Pregunta 17.- R: 3
hipoxia. La lesión de la matriz germinal produce hemorragia de La ictericia por lactancia materna (LM) es la causa más frecuente de ic-
la misma que se diagnostica a través de una ecografía cerebral tericia tardía. Su frecuencia es de aproximadamente 1/200 (R1).
(respuesta 3). Se debe a que en la leche materna existen unas sustancias (pregna-
nodiol, ácidos grasos de cadena larga) que inhiben la glucoronil
Pregunta 14.- R: 4 transferasa, dando como resultado un aumento de la bilirrubina
Estamos ante un RN con un cuadro de enterocolitis necrotizante (NEC). a expensas de fracción indirecta (La respuesta 4 es, por lo tanto,
La presentación típica es en un RNPT que a la semana inicia cuadro correcta).
de vómitos, distensión abdominal y deposiciones con sangre. El En ocasiones, cuando existen dudas diagnósticas, se puede inte-
primer diagnóstico que nos debemos plantear es una NEC. rrumpir la LM transitoriamente durante unos días, observándose
Dentro de los factores de riesgo figuran, aparte de la prematuridad, un claro descenso en la bilirrubina, pero ésta no es indicación de
la administración precoz y en altas concentraciones de leche y suspender definitivamente la LM, puesto que no se han descrito
factores infecciosos. casos de kernicterus (Respuesta 3).
La prueba diagnóstica indicada sería una Rx de abdomen donde se
apreciaría el dato más característico, la neumatosis intestinal. Pregunta 18.- R: 5
El tratamiento se debe instaurar ante la mínima sospecha, puesto Para el MIR es importante saberse bien las diferencias entre la in-
que de eso va a depender el pronóstico. compatibilidad Rh y ABO.
Si el niño no tiene signos de perforación intestinal, se dejará a dieta
absoluta durante semanas, con antibioterapia. Si en algún mo-
Rh AB0
mento de la evolución se produjera una perforación intestinal o
sepsis refractaria a tratamiento antibiótico, sería necesario realizar • Madre Rh- Hijo Rh+.
• Madre 0 Hijo A.
tratamiento quirúrgico. • Necesaria inmunización previa.
• Puede producirse en el primer
• La incompatibilidad de grupo protege
Las complicaciones a largo plazo son la estenosis intestinales y el embarazo.
frente a la incompatibilidad Rh.
síndrome de intestino corto. • No se agrava con los siguientes.
• Se agrava en los siguientes embarazos.
• Ictericia. • Ictericia.
Pregunta 15.- R: 3 • Anemia. • Anemia.
El cuadro clínico se refiere a un íleo meconial. La forma de presentación • Hidrops fetal: en casos graves. • No hidrops.
más habitual en los neonatos es como un cuadro de obstrucción
• Prenatal: no tienen utilidad.
intestinal. El dato que apoyaría este diagnostico sería palpar unos • Prenatal: CI a la madre durante 1T y 3T.+
• Postanatal: CD+ débil, CI+++,
cordones duros que siguen el marco cólico, puesto que el lugar • Postnatal: CD+++.
con esferocitosis.
donde con mayor frecuencia se impacta el meconio es la zona de
íleon distal y colón proximal. Tratamiento: fototerapia, exanguino-
Idem
transfusión.
Una vez establecido el diagnóstico es siempre obligatorio descartar
una fibrosis quística (un 15% debutan como íleo meconial) con la Prevención: Gamma en 3T y en las prime-
realización de un tripsinógeno sérico, que estará aumentado. ras 72 horas tras el parto si la madre tiene No tiene
un test de CI negativo.
El tratamiento, si el niño no tiene signos de perforación intestinal,
se realiza mediante la administración de enemas hiperosmolares. Pregunta 18. Diferencias entre incompatibilidad Rh y AB0.
Si con esta medida no se soluciona el problema, sería necesario
recurrir a la cirugía, realizando una resección de la zona de impac- Pregunta 19.- R: 4
tación y anastomosis termino-terminal. El caso clínico hace referencia a una enfermedad hemorrágica del RN. La
causa es un déficit de vitamina K, debido a que durante el embarazo
Pregunta 16.- R: 3 el paso transplacentario de vitamina es escaso, la LM es pobre en
Respecto a las ictericias neonatales, es importante tener claro varios vitamina K (la opción 4 es falsa) y en el intestino no hay bacterias que
aspectos. sinteticen vitamina. Todas estas circunstancias obligan a administrar

CTO Medicina • C/Nuñez de Balboa 115 • 28006 Madrid • Tfno. (0034) 91 782 43 30/33/34 • E-mail: secretaria@ctomedicina.com • www. ctomedicina.com 3

Willy Aguilar
Comentarios de Test a distancia 1ª vuelta
Pediatría
vitamina K (1 mg im ) a todos los RN en las primeras 24 horas de vida. hijos de madres con diabetes gestacional).Se debe a que el HMD
En algunas ocasiones va a ser necesario más de una dosis. sintetiza exceso de insulina para compensar el déficit materno
La clínica consiste en sangrado a distintos niveles: umbilical, diges- de esta hormona. Suele aparecer en las primeras horas de vida y
tivo, nasal,... normalmente cursa de forma asintomática.
El tratamiento consiste en la administración de nuevas dosis de El control durante el parto de la glucemia materna junto con el inicio
vitamina K o plasma fresco congelado. precoz de la alimentación oral (respuesta 5 falsa), disminuyen el
Es importante recordar que los hijos de madres que durante el em- riesgo de hipoglucemia neonatal.
barazo han tomado fenitoína o fenobarbital tienen más riesgo de
padecer este trastorno.
Pregunta 23.- R: 1
Pregunta 20.- R: 1 El embarazo del hijo de madre diabética es considerado un embarazo
La llamada anemia fisiológica se debe a un déficit transitorio de EPO, de alto riesgo, debido al gran número de problemas que puede
que unido a la hemólisis de glóbulos rojos durante los primeros presentar el RN. Entre ellos figuran:
meses de vida, dejan al niño anémico. • Macrosomía con visceromegalia (pero no megalencefalia), excepto
Las cifras más bajas se producen a los 2 meses en los RNPT y a los 3 las madres con diabetes avanzadas, y vasculopatía, que suelen
en los RNT (R1). tener niños CIR.
El tratamiento consiste en suplementos de hierro a partir de los 2 • Cardiomiopatías: lo más frecuente es la hipertrofia asimétrica del
meses en los RNPT, ya que antes los depósitos están llenos por tabique interventricular, que clínicamente se manifiesta como una
la hemólisis de los glóbulos rojos, y en casos seleccionados sería estenosis subaórtica.
necesario la trasfusión de concentrados de hematíes. • Síndrome asfíctico: son niños más grandes y con más complica-
En los neonatos no se manifiesta la betatalasemia, porque la cadena beta ciones durante el parto.
de la hemoglobina se empieza a sintetizar a partir de los 6 meses. • Riesgo de EMH: la insulina en altas concentraciones inhibe la
síntesis de surfactante.
Pregunta 21.- R: 3 • Policitemia, ictericia y trombosis de la vena renal.
Es muy importante recordar las situaciones en las que se debe • Malformaciones: las más frecuentes malformaciones cardíacas.
administrar gammaglobulina específica anti-D para prevenir la • Otras: agenesia lumbosacra y sd. del colón izquierdo hipoplási-
isoinmunización Rh. Es necesario inyectar la gammaglobulina co.
anti-D a las 28 semanas de gestación y en las primeras 72 horas • Alteraciones metabólicas:
después del parto, aborto o amniocentesis, si se confirma que el - Hipoglucemia: más frecuente en las primeras horas. Es necesario
recién nacido es Rh positivo y la madre Rh negativo. La profilaxis un control (venoso o capilar) pues la mayoría de las veces es
se hará sólo si la gestante no ha sido previamente sensibililzada, asintomático. El tratamiento consiste en el aporte de glucosa iv
es decir, sólo si el test de Coombs indirecto de la gestante es ne- mediante perfusión continua, evitando los bolos por el riesgo de
gativo (respuesta 3 falsa). hiperglucemia de rebote.
- Hipocalcemia.
Pregunta 22.- R: 5
La hipoglucemia neonatal es bastante frecuente en los hijos de madre Pregunta 24.- R: 3
diabética (75% de los hijos de madres diabéticas y 25% de los Ver el esquema «Características de la sepsis neonatal».
Pregunta 24. Características de la sepsis neonatal.

Características Comienzo precoz. Comienzo tardío. Nosocomial.


Nacimiento; menos de 7 días 8-28 días, en ocasiones hasta De la primera semana al alta Hospita-
Comienzo
normalmente menos de 3 días. 60 días. laria.
Poco frecuentes. Premadurez; ingreso en UCI
Riesgos obstétricos Colonización, amnionitis,
premadurez. neonatales, resección intestinal.
Dificultad respiratoria, neumonía Fiebre, signos nerviosos centrales Apnea, bradicardia, letargia,
Presentación.
Shock. o focales. inestabilidad térmica.
Meningitis 20%. 75%. 10-20%.
Neumonía, pielonefritis, endoftalmitis,
trombos sépticos, flebitis, infecciones
Pielonefritis, osteomielitis, artritis
Otros sistemas. Raro.
séptica, celulitis.
cutáneas, sepsis de líneas centrales,
enterocolitis necrotizante.
Microorganismos
Estreptococo del grupo B tipos
Staphylococcus epidermidis, S.
Ia, Ib, Ia/c, II, III, E. coli,
Estreptococo del grupo B tipo III, aureus,patógenos.
Klebsiella, Listeria monocytogenes
patógenos. E. coli Ag K1 L. monocytogenes, Candida albicans, Pseudomonas aerugino-
enterococos, Haemophilus
herpes simple. sa, E. coli, herpes simple, Klebsiella,
influenzae no tipables,
Serratia.
S. pneumoniae.
Depende de los agentes nosocomiales
Ampicilina y gentamicina o Ampicilina y gentamicina o
Tratamiento. Presentes en la sala de neonatología;
cefotaxima. cefotaxima.
cefotaxima, vancomicina y gentamicina.
Ventilación mecánica, fármacos Ventilación mecánica, fármacos Ventilación mecánica, fármacos
Medidas de apoyo. vasoactivos, reposición vasoactivos, reposición vasoactivos, reposición
hidroelectrolítica, ECMO. hidroelectrolítica. hidroelectrolítica.
Mortalidad. 15-70%. 10-20%. hidroelectrolítica.

CTO Medicina • C/Nuñez de Balboa 115 • 28006 Madrid • Tfno. (0034) 91 782 43 30/33/34 • E-mail: secretaria@ctomedicina.com • www. ctomedicina.com 4

Willy Aguilar
Comentarios de Test a distancia 1ª vuelta
Pediatría
IgGCTOMEGALOVIRUS TORMOPLAMA
• Se reactiva Jasintomática 1-2%
Infección materna • Se infecta
• Se infecta Jsintomática 1-2%0
• Placentaria 1º trimestre más frecuente, más grave • Placentario
Transmisión • Canal de parto 1º trimestre más grave
• Lactancia 3º trimestre más frecuente
• 5-18% secuelas tardías, hipoacusia neurosensorial bilateral
Ansiolítico más frecuente Sin tratamiento Jcoriorretinitis
y severa.
SABIN
• Corriorretinitis • Coriorretinitis
Síntoma • Microcefalia • Hidrocefalia
• Calcificaciones periventriculares • Calcificaciones intracraneales difusas
• Convulsiones
1ª determinación:
• Negativo: vigilar
Screening prenatal • Prenatal Ac antiCMV (no se hace prevención) • Positivo: hacer una 2ª determinación:
- Disminuido igual: infección pasada
- Aumantado: tratar
• Aislamiento y cultivo en orina “inclusiones en ojo de buho” • Aislamiento en placenta (sangre?)
Diagnóstico • KIgM • KIgM
• KIgG o estable a las 6 semanas • KIgG o estable a las 6 semanas
Siempre:
No • Sintomático: primetadina + sulfadicina 6 meses
Tratamiento
• Ganciclovir a veces primetadina + sulfamicina/espiromicina 6 meses
• Asintomático: espiromicina y valorar serología
Pregunta 27. Diferencias entre toxoplasma y citomegalovirus.

Pregunta 25.- R: 5
La madre que puede contagiar una hepatitis B al RN es aquella que
tiene el Ag s +, (hepatitis crónica activa, hepatitis aguda, portadora).
Si además el Age es positivo, el riesgo asciende hasta un 90%.
El momento de mayor riesgo es el momento del parto.
La mayoría de las veces, la enfermedad en el neonato es asintomática,
pero tiene una alta probabilidad de evolucionar hacia una forma
crónica y sufrir degeneración maligna.
Para evitar este curso es fundamental realizar profilaxis con vacuna
frente a hepatitis B (0,1,6) y gammaglobulina específica en el
mismo momento. Si la profilaxis se realiza correctamente, podrá
recibir lactancia materna.

Pregunta 26.- R: 2
El caso clínico nos presenta a un RN con clínica de CMV congénita.
Los datos claves que nos permiten llegar al diagnóstico son:
coriorretinitis y calcificaciones periventriculares (en el caso de
la TXP, estas son difusas). La infección congénita por CMV es la
infección congénita más frecuente. Lo más frecuente es que estén
Pregunta 28. Manifestaciones clínicas de la rubéola congénita.
asintomáticos, pero si tienen clínica pueden presentar ictericia,
hepatoesplenomegalia, calcificaciones, hepatitis,... La neumonitis
Aparte de esto, para el MIR debemos recordar:
es la forma más característica de infección postnatal. No existe
• Al contrario que en todas las infecciones connatales, la rubéola
tratamiento eficaz; en casos de alta replicación al nacimiento
tiene más riesgo de trasmisión en el 1º trimestre.
se puede emplear Ganciclovir. La secuela más frecuente es la
• La tríada de Gregg se caracteriza por:
hipoacusia neurosensorial.
- Sordera de percepción.
- Cardiopatía: la más frecuente el DAP.
Pregunta 27.- R: 2 - Afectación ocular: lo más frecuente son las cataratas.
Ver tabla «Diferencias entre toxoplasmosis y citomegalovirus». • Un RN infectado por rubéola congénita puede eliminar el virus en
las secreciones hasta año y medio después del nacimiento, por lo
Pregunta 28.- R: 1 que es obligatorio aislarlo de mujeres embarazadas.
Estamos ante un niño RN CIR (edad gestacional de 38 semanas y peso <
2500gr) con clínica de erupción petequial y hepatoesplenomegalia. Pregunta 29.- R: 1
Esto lo puede presentar cualquier infección connatal. De los factores que se citan el más relacionado con el CIR tipo simétrico
A continuación nos describen la clínica típica del DAP: soplo continuo (CIT tipo I) es la infección congénita.
en 2º espacio intercostal izquierdo con pulsos femorales saltones. Las Recuerda que hay que sospechar una infección connatal ante la pre-
lesiones óseas son las típicas de la rubéola, las que nos encontraría- sencia de CIR tipo simétrico, hepatoesplenomegalia, adenopatías,
mos en la sífilis serían las mismas, pero con reacción perióstica. ictericia, anemia y trombopenia en un recién nacido.

CTO Medicina • C/Nuñez de Balboa 115 • 28006 Madrid • Tfno. (0034) 91 782 43 30/33/34 • E-mail: secretaria@ctomedicina.com • www. ctomedicina.com 5

Willy Aguilar
Comentarios de Test a distancia 1ª vuelta
Pediatría
Pregunta 30.- R: 4
Todo el caso clínico hace referencia a la infección por el virus herpes El granuloma, que aparece al caerse el cordón, es un tejido blando,
simple, siendo el más frecuente el tipo II. granular, vascular y rojizo o rosado, que puede tener a veces una
(Ver tabla «Infecciones connatales»). secreción mucopurulenta. Otras alteraciones son el onfalocele (dd
La clínica suele comenzar a la semana con grave afectación del estado con gastrosquisis) y la hernia umbilical.
general, fontanela abombada y vesículas en la zona de presentación
(en la práctica clínica sólo aparecen en un 70% de los casos). Se Sobre secreciones por el ombligo han Preguntando en el
asocia con una elevada mortalidad. MIR:
El neonato contrae la infección cuando pasa por el canal del parto, • Persistencia del uraco: sale por el ombligo un líquido amarillento
por lo tanto lesiones genitales activas en el momento del parto similar a la orina con un pH ácido.
contraindican un parto vaginal y es indicación de cesárea. • Persistencia del conducto onfalomesentérico (fístula intestinal):
El tratamiento en este niño sería la administración de aciclovir in- sale un líquido amarillento con un pH alcalino.
travenoso. El ombligo amniótico se refiere a un ombligo con mucha gelatina.

Pregunta 31.- R: 2 Pregunta 32.- R: 4


El hipotiroidismo congénito es una patología relativamente común
Onfalo- que se debe descartar en todos los neonatos. Su causa más fre-
Hernia umbilical Gastrosquisis
cele cuente es la disgenesia tiroidea.
DEFECTO DE Paramedio En el recién nacido los signos y síntomas de hipotiroidismo son muy
Umbilical. Umbilical. sutiles (respuesta falsa: 4) y la clínica se va instaurando de forma
CIERRE (lat. dcho. + frec.).
progresiva. Aparece facies peculiar, estreñimiento, ictericia prolon-
CUBIERTAS Peritoneo y piel. Peritoneo. Sin peritoneo.
gada, letargia, hernia umbilical, fontanelas amplias, retraso en la
- Anomalías cro- maduración ósea y puede asociarse una sordera neurosensorial.
- Estrangulación mosómicas. La determinación de TSH en sangre, obtenida entre los 2 y los 5 días
COMPLICA- (rara). - Extrofia vesical.
Atresia intestinal. de vida, es la prueba de sreeening rutinaria. Cuando se confirma
CIONES - Reducción - Sd. Beckwith
espontánea. (macrosomía e el diagnóstico es necesario iniciar el tratamiento precoz con levo-
hipoglucemia). tiroxina para evitar la aparición de retraso mental irreversible.
Quirúrgico, si:
- Estrangulación. Correción quirúr- Pregunta 33.- R: 4
Correción quirúrgi- Recordar los siguientes datos de forma muy simplificada:
- Crecimiento gica precoz
TRATAMIENTO ca precoz • 1,5 m: sonrisa social.
progresivo. (buen pronós-
(peor pronóstico).
- Persiste a los tico). • 3 m: inicio sostén cefálico.
3-5 años. • 6 m: inicia sedestación.
Pregunta 31. Patología umbilical. • 9 m: movimiento de pinza.
Pregunta 30. Infecciones connatales.
VARICELA SÍFILIS RUBEOLA HERPES SIMPLE
• Se reactiva (VHS II)
Infección materna Se infecta Se infecta Se infecta Jsintomática 70%
• Se infecta: poco frecuente
Placentaria:
Placentaria: 3º trimestre: más • Placenteria
• 1º y 2º trimestre, más Placentaria 1º trimestre más
Transmisión
frecuete
frecuente si madre con sífilis pri-
frecuente, más greve
• Canal del parto
mariA o secundaria no tratadas • Postparto
• Próximo a parto: más grave
• Transmisión 1º y 2º trimestre:
atrofia miembros, cicatrices y
• Proecoz (<de 2 años) • Transitoria: CIR, hepatoesple- • Generalizada (1ª semana)
Hepatomegalia. nomegalia, meningitis... sepsis, encefalitis...
malformaciones.
Pénfigo sifilítico, coriza, neumo- • Permanente (Gregg): • Localizada
• 5 a 21 días preparto: varicela nía alta de Hetcht
Síntomas Cataratas (2ª y 3ª semanas):
leve en 4 primeros días pos-
tparto.
• Tardía (>de 2 años) Sordera e percepción - Queratoconjuntivitis
Sordera de percepción Cardiopatía -Vesículas cutáneas
• 5 días preparto a 2 días pos- Queratitis intersticial Retraso mental, fenómenos -Encefalitis del lóbulo
tparto: varicela grave en días 5
Anomalías dentarias autoinmunes temporal
y 10 postparto.
• 1º trimestre y antes del parto
Sreaning prenatal -
• VDRL y FTA-ABS. - -
• Si infección activa tratar a la
madre
• ADN del virus
• IgM e sangre de cordón • Aislamiento de virus
• VDRL
Diagnóstico (descenso rápido). • KIgm Cultivo y aislamiento virus
• FTA-ABS Jtratar con penicilina
• Aislamiento de virus en el • KIgG o estable a la 6 semanas
líquido vesicular.
Si infección materna en 5 días
preparto y 2 postparto:
Tratamiento • Ig anti VVZ Penicilina 15 días No Aciclovir
• Aislamiento estricto
• Aciclovir si es greve

CTO Medicina • C/Nuñez de Balboa 115 • 28006 Madrid • Tfno. (0034) 91 782 43 30/33/34 • E-mail: secretaria@ctomedicina.com • www. ctomedicina.com 6

Willy Aguilar
Comentarios de Test a distancia 1ª vuelta
Pediatría
• 10-12 m. Inicia bipedestación.
• 12 m: emite bisílabos referenciales. La causa más frecuente de talla baja en el niño son las dos variantes
• 15-22 m: realiza torres de cubos de 2 hasta seis. de la normalidad.
• 16-19 m: corre y realiza combinaciones de 2 palabras.
Pregunta 38.- R: 5
Pregunta 34.- R: 5 Si tenemos presente la tabla del comentario de la pregunta anterior
Pregunta que mezcla aspectos del desarrollo ponderoestatural y podemos deducir fácilmente que estamos ante un caso de talla
psicomotor durante los primeros meses de vida. baja familiar. La curva de crecimiento es inferior al percentil 3, no
Recuerda que el peso del niño se duplica al 5º mes, se triplica al año hay discordancia entre la edad ósea del niño y su edad cronológica
y se cuadriplica a los 2 años. La talla del nacimiento se duplica y, además, existe historia familiar de talla baja. Las pruebas de la-
alrededor de los 4 años de edad (respuesta 5 falsa). boratorio serán normales y la talla adulta final esperable será baja,
pero dentro de los límites esperados para su talla genética.
Pregunta 35.- R: 2
El calostro es la leche de los 2-4 primeros días después del parto. Su Pregunta 39.- R: 3
densidad es mayor que la leche humana madura pues contiene Este es un cuadro de laringitis aguda caracterizado por un antecedente
mayor cantidad de proteínas y minerales que ésta (respuesta previo de CVA con fiebre de 38-38.5º y tos perruna.
2 falsa), y una serie de factores inmunitarios importantes en la El tratamiento consiste en humedad ambiental, corticoides (en aerosol
defensa del RN. o sistémicos) y adrenalina en aerosol.
Con los días el calostro es sustituido por una leche de transición que
se convierte en madura hacia la 3ª-4ª semana. Pregunta 40.- R: 2
Ver cuadro «Crup infeccioso».
Pregunta 36.- R: 2
Pregunta 41.- R: 3
LECHE HUMANA LECHE DE VACA La epiglotitis es una enfermedad típica de los niños menores de 5 años;
Calorías 670 kcal/l 670 kcal/l en la actualidad los principales responsables de esta entidad son
los cocos gram positivos como el S. pyogenes, S. pneumoniae y S.
1-1,5 g% 3-4,5 g%
aureus. Por detrás de éstos, hay que tener en cuenta también a su
Proteínas Caseína 30% Caseína 80%
Seroproteínas 70% Seroproteínas 20%
agente clásico, el H. influenzae tipo B (cada vez menos frecuente
gracias a la vacunación universal).
Hidratos de La clínica suele cursar de forma brusca, por la noche, acompañándose
7 g% lactosa y otras 4,5 g% lactosa
Carbono
de intensa dificultad respiratoria y apenas sin tos, con fiebre alta
3,5 g% y aspecto séptico.
Ác. grasos esenciales 3,5 g% En el tratamiento, lo prioritario es asegurarse la vía aérea (mediante
Grasas Ác. grasos cadena larga Escasos ác. grasos esenciales intubación orotraqueal o traqueostomía) y administración de
insaturados Ác. grasos saturados
antibióticos, generalmente cefalosporinas de 3ª generación. Con
Colesterol
el tratamiento adecuado, el cuadro suele remitir en 24–48 horas.
Minerales + +++ (5 veces) En los casos dudosos nunca hay que intentar visualizar la epiglotis,
Hierro + + sino realizar una Rx lateral de faringe donde se apreciará una
Cobre ++ + epiglotis engrosada.
Flúor + –
Pregunta 42.- R: 5
Relación Cál- Estamos ante un cuadro de bronquiolitis aguda. El agente más fre-
2 1
cio/Fósforo cuente es el VRS. Es una enfermedad estacional de los meses de
Vitamina A ++ + invierno y primavera.
Vitamina B + ++ Afecta a niños menores de 2 años, principalmente lactantes. Comienza
por un cuadro catarral, con tos blanda, mocos y febrícula, y en
Vitamina C + Escasa
los días siguientes evoluciona hacia un cuadro de obstrucción de
Vitamina D + Escasa vías aéreas distales con sibilancias y espiración alargada. Esta fase
Vitamina E ++ + crítica suele durar 2- 3 días con posterior mejoría.
El tratamiento consiste en fisioterapia y beta 2 agonistas inhalados
Vitamina K Escasa +
(terbutalina, adrenalina,...).
Nitrógeno no La ribavirina estaría indicada en niños con patología pulmonar o
+++ +
protéico cardiaca de base; ni los corticoides ni los antibióticos estarían
Pregunta 36. Diferencias entre la leche materna y la de vaca. indicados. Actualmente se intenta prevenir su aparición adminis-
trando gammaglobulina específica mensualmente en los meses
Pregunta 37.- R: 3 de riesgo a los lactantes con patología de base (cardiópatas,
neumópatas).
Caracterís- La secuela a largo plazo más frecuente es la hiperreactividad bronquial.
Talla baja familiar Retraso constitucional
ticas
Causas Talla baja en los padres Antecedentes familiares Pregunta 43.- R: 4
El cuadro que presenta este niño es claramente una fibrosis quís-
• Velocidad crecimiento • Velocidad crecimiento lenta. tica.
normal. • Pubertad retrasada.
Diagnóstico Las manifestaciones clínicas suelen ser:
• EO= EC <E talla. • EO= E talla <EC.
• Talla final= talla diana. • Talla final= talla diana. • Respiratorias: bronquiolitis recurrentes, tos crónica, neumonía,
pólipos nasales, sinusitis,...
Tratamiento No precisa No precisa
• Digestivas: íleo meconial, esteatorrea, malabsorción, prolapso
Pregunta 88. Tipos de talla baja no patológica. rectal, cirrosis biliar,...

CTO Medicina • C/Nuñez de Balboa 115 • 28006 Madrid • Tfno. (0034) 91 782 43 30/33/34 • E-mail: secretaria@ctomedicina.com • www. ctomedicina.com 7

Willy Aguilar
Comentarios de Test a distancia 1ª vuelta
Pediatría
Pregunta 40. Crup infeccioso.
LARINGITIS AGUDA (estridulosa) LARINGOTRAQUEÍTIS VIRAL EPIGLOTITIS AGUDA
Etiología Alergia + psicológico Virus parainfluenzae 1 (el más frecuente) H. influenzae b
Antecedentes No hay Catarro vías altas (paciente y / o familia) --
Espasmo laríngeo recortado
Fiebre alta + babeo + disfagia + disnea +
Clínica (generalmente nocturno). No Fiebre, tos, disnea alta, estridor inspiratorio
cabeza extendida
fiebre
Duración 1-2 noches Días - semanas. Fulminante
• Ambiente tranquilo, humidificar. • Intubar + oxígeno.
Tratamiento • Corticoides. • Cefalosporina de 3ª generación.
• Adrenalina racémica en aerosol. • Tto. en UVI.

• Otras: diabetes, azoospermia.


Para el diagnóstico se requiere al menos dos test positivos del sudor, Pregunta 47.- R: 4
junto con manifestaciones clínicas compatibles y/o presencia de La causa más frecuente de GEA en nuestro medio es el rotavirus.
mutaciones en el estudio genético. El estudio genético no puede Es una enfermedad típica de lactantes que produce una diarrea
desplazar al test del sudo debido a la gran variedad de mutaciones líquida, SIN sangre. El diagnóstico se basa en la realización de un
que existen. test de ELISA en las heces para detectar el Ag.
Para el tratamiento de estos pacientes es fundamental el cuidado El tratamiento es sintomático.
del pulmón. Se debe realizar diariamente fisioterapia respiratoria
y tratamiento agresivo de las infecciones pulmonares. Pregunta 48.- R: 1
Estamos ante un cuadro de alergia a proteínas de leche de vaca. El
Pregunta 44.- R: 1 cuadro típico consiste en inicio de la clínica tras la ingesta de un
La traqueítis bacteriana tiene como principal responsable al S. aureus biberón de leche de vaca. La sintomatología suele ser digestiva
(respuesta 1 falsa). Hay que sospechar esta entidad cuando, tras (vómitos, diarrea), cutánea (urticaria) y en ocasiones respiratoria
un cuadro de crup vírico, se produce un empeoramiento progre- (broncoespasmo, edema de glotis).
sivo con fiebre, dificultad respiratoria de intensidad creciente y El cuadro está mediado por un mecanismo inmunológico tipo Ig E.
estridor mixto. Para el diagnóstico se emplean las pruebas cutáneas, que serán
Es una entidad grave que suele requerir la hospitalización del niño positivas.
para instaurar tratamiento antibiótico (Cloxacilina i.v) y oxigeno- El tratamiento consiste en la suspensión de la leche de vaca, reempla-
terapia a demanda. Si aparece gran dificultad respiratoria puede zándola por fórmulas especiales (hidrolizado de proteínas). Suele
ser necesaria la intubación. ser un proceso autolimitado, la mayoría de los niños pueden tomar
leche de vaca a partir de los 2 años.
Pregunta 45.- R: 4
En la patogenia de la fibrosis quística está implicada una alteración de Pregunta 49.- R: 4
la regulación de los canales iónicos de las membranas celulares, lo Caso clínico de atresia con fístula traqueoesofágica. Según la clasi-
que produce secreciones deshidratadas y espesas. A nivel pulmonar, ficación de Ladd, el tipo más frecuente es la atresia proximal con
la lesión anatomopatológica inicial es la bronquiolitis. fístula distal (tipo III).
El íleo meconial supone la forma de debut neonatal en aproximada- El diagnóstico se realiza por la sospecha clínica (salivación excesiva,
mente un 10% de los casos. La función endocrina del páncreas se cianosis y atragantamiento con las tomas,…) e imposibilidad para
afecta con el tiempo, pudiendo aparecer Diabetes Mellitus a partir pasar una sonda nasogástrica.
de los 10 años de evolución de la enfermedad. El tratamiento es quirúrgico, siendo el reflujo gastroesofágico la
Debido a la pérdida excesiva de sal, se ven cuadros de deshidratación complicación postquirúrgica más frecuente, que suele ser grave
con hiponatremia e hipocloremia, coincidiendo con gastroenteritis (respuesta 4). Otras complicaciones menos frecuentes son la fís-
o en épocas de calor. tula de la anastomosis, la recidiva de la fístula traqueoesofágica,
La aspergilosis broncopulmonar alérgica es relativamente frecuente la estenosis esofágica y la traqueomalacia.
en estos pacientes. Hay que sospecharla ante la presencia de
esputo herrumbroso, aislamiento de Aspergillus fumigatus o Pregunta 50.- R: 5
la presencia de eosinófilos en una muestra fresca de esputo. El Estamos ante un caso de RGE leve, caracterizado por pequeños
tratamiento será corticoterapia oral, en los casos refractarios vómitos sin fuerza después de las tomas, por lo demás el niño
puede ser necesario el empleo de anfotericina B en aerosol o de se encuentra asintomático con peso y talla en percentiles nor-
5-fluorocitosina sistémica. males. El RGE es muy frecuente en el primer año de vida, hasta
un 85%. Generalmente desaparece hacia los 2 años, aunque en
Pregunta 46.- R: 4 ocasiones puede persistir hasta los 4.
Es necesario iniciar el tratamiento de la fibrosis quística antes de En un 10% de los casos, el RGE se asocia con escasa ganancia
que la afectación pulmonar sea importante. Se ha visto que el ponderal, esofagitis, anemia ferropénica y clínica respira-
empleo de fisioterapia respiratoria alarga la supervivencia de toria.
estos pacientes. El diagnostico se basa en la historia clínica y en ocasiones en la
Es útil el empleo moderado de enzimas pancreáticas para disminuir realización de pruebas complementarias: transito superior con
la esteatorrea, así como el aporte de suplementos de vitaminas bario y una pHmetría de 24 horas (para cuantificar el grado de
liposolubles a la dieta. reflujo, es la prueba más sensible y específica).
En los pacientes con fibrosis quística, la neumonía por sobreinfección El tratamiento en los casos leves consiste en medidas posturales
bacteriana se debe principalmente a Pseudomonas aeruginosa, y espesantes de las tomas. En casos de reflujos más graves
variedad mucoide. Este patógeno puede colonizar la vía respirato- se administraran estimulantes del peristaltismo (cisapride,
ria dando lugar a un estado de portador crónico difícil de revertir domperidona) o tratamiento quirúrgico (funduplicatura de
(respuesta falsa: 4). Nissen).

CTO Medicina • C/Nuñez de Balboa 115 • 28006 Madrid • Tfno. (0034) 91 782 43 30/33/34 • E-mail: secretaria@ctomedicina.com • www. ctomedicina.com 8

Willy Aguilar
Comentarios de Test a distancia 1ª vuelta
Pediatría
tratamiento con enemas de bario o con aire, para intentar la
Pregunta 51.- R: 5 reducción.
La clínica típica de un cuadro de estenosis hipertrófica de píloro Un 5% puede recidivar (frecuente en aquellos casos en los que se
consiste en vómitos pr oyectivos tras las tomas, tras el vómito realiza reducción hidrostática).
el niño se queda irritable y hambriento (ver tabla «Vómitos del
lactante»). Pregunta 55.- R: 4
Debido a los vómitos persistentes, es típico encontrar una alcalosis La enfermedad de Hirschprung se caracteriza por estreñimiento
metabólica con normo o hipo K. crónico desde el nacimiento asociado a un estancamiento o
La prueba diagnóstica a realizar ante la sospecha de este cuadro retraso ponderal (por malabsorción).
sería una ecografía abdominal. En ocasiones se debe recurrir a Para el diagnóstico es útil el tacto rectal, donde se aprecia una
un tránsito digestivo, donde aparece de forma típica la imagen ampolla rectal vacía de heces. En la manometría se aprecia un
del “signo de la cuerda”. aumento paradójico del tono del esfínter anal. El diagnóstico
El tratamiento es quirúrgico: pilorotomía extramucosa de Rams- definitivo lo dará la biopsia: en el segmento afectado se observa
ted. ausencia de células ganglionares con aumento de la acetilcoli-
nesterasa y aumento de las terminaciones nerviosas.
Pregunta 52.- R: 4
Lo típico de la atresia duodenal son los vómitos biliosos en las Pregunta 56.- R: 5
primeras 24- 48 horas, con abdomen excavado. Con mayor Los factores que intervienen en la patogenia de la celiaca son:
frecuencia la atresia se localiza a nivel de la 3ª porción del • HLA: DR3, DR4, DR7 , DQ y W2.
duodeno. En la Rx de abdomen es típico encontrar la imagen • Factores inmunológicos: los linfocitos de la lámina propia se
de “ doble burbuja”. inmunizan frente a la gliadina.
La R1: lo típico sería encontrar vómitos a las 2-3 semanas de • Factores ambientales.
vida.
R2: lo típico es estreñimiento y en ocasiones vómitos fecaloi-
deos.
R3: sangrado indoloro rectal sin vómitos.
R5: la clínica de la invaginación intestinal suele comenzar con
episodios de dolor intenso, siendo muy raro en el periodo
neonatal.

Pregunta 53.- R: 4
El divertículo de Meckel es un resto del conducto onfalomesentérico.
Es la malformación digestiva más frecuente (R1), localizándose
a unos 50- 70 cm de la válvula ileocecal.
La clínica más frecuente suele consistir en una hemorragia rectal
indolora (R4) e intermitente debido a la ulceración de la mucosa
ileal adyacente al divertículo con mucosa ectópica (gástrica o
pancreática).
La técnica diagnóstica más sensible es la gammagrafía con Tc 99.
El tratamiento es quirúrgico.

Pregunta 54.- R: 3
Estamos ante un cuadro típico de invaginación intestinal. Niño
que presenta episodios repetidos de llanto y encogimiento de
piernas con posterior decaimiento. Si la clínica evoluciona hasta
un 60%, puede presentar deposiciones en jalea de grosella,
heces con sangre roja fresca y moco.
Para el diagnóstico inicial se suele realizar una ecografía abdo-
Pregunta 56. Diagnóstico de enfermedad celíaca.
minal.
Una invaginación intestinal es siempre una urgencia. Si han tras-
La clínica se caracteriza por un cuadro de malabsorción, de inicio
currido < de 48 horas y no hay signos de perforación, se realiza
más frecuente entre los 6 meses y los 2 años. Suelen presentar
Pregunta 51. Vómitos del lactante.
Reflujo gastroesofágico Estenosis hipertrofica de píloro Altresia pilórica Atresia duodenal
Momento de aparición 1ª semana 2ª a 3ª semana
Vómitos Alimentarios sin fuerza Alimentarios a chorro Alimentarios Biliosos
• Síntomas respiratorios. • Masa palpable • Abdomen excavado
Otros síntomas
• Disminución del crecimiento • Alcalosis hipoclorémica • Asociar al Down
Clínica + respuesta a tratamiento
L Evolución. RX: RX:
Diagnóstico pHmetría 24h • Rx: “signo de la cuerda” • 1 burbuja gástrica • 1 burbuja gástrica
L • 1 burbuja gástrica • Ausencia de gas distal • 1 burbuja duodenal
encoscopia
• Medidas posturales Cirugía:
Tratamiento • Cisapride • Piloromiotomía Cirugía Cirugía
• Cirugía (fundoplicatura de Nissen) • Fredet Ramsted

CTO Medicina • C/Nuñez de Balboa 115 • 28006 Madrid • Tfno. (0034) 91 782 43 30/33/34 • E-mail: secretaria@ctomedicina.com • www. ctomedicina.com 9

Willy Aguilar
Comentarios de Test a distancia 1ª vuelta
Pediatría

esteatorrea, estancamiento de la curva ponderal, disminución Pregunta 59.- R: 3


de la masa muscular, irritabilidad y distensión abdominal im- Es un cu adro de malabsorción a hidratos de carbono. Tiene como
portante. antecedente una GEA que lesiona el borde del enterocito, produ-
El diagnóstico se basa en los test serológicos (Acs antigliadina, anti- ciendo un déficit transitorio de lactasa. La clínica se caracteriza
rreticulina, antiendomisio y antitransglutaminasa, estos últimos por diarrea, dolor cólico, heces espumosas y ácidas (pH< 5.5) que
los más sensibles y específicos de todos, y en la realización de excorian la zona del pañal. Ante la sospecha se debe realizar un
biopsias intestinales. Clinitest en las heces que será positivo. El tratamiento consiste en
El tratamiento consiste en la exclusión de por vida del trigo, la exclusión de la lactosa de la dieta, que la mayoría de las veces
cebada, centeno y +/- avena. Pueden tomar libremente es transitoria.
maíz y arroz. La R1 es falsa porque, aunque la clínica es igual, el Clinitest es negativo.
La R2: no da clínica. La R4: produce una malabsorción de proteínas
Pregunta 57.- R: 2 con clínica de desnutrición y edemas.
Pregunta importante sobre el diagnóstico de la enfermedad celíaca. R5: la clínica del rotavirus es vómitos y diarrea líquida, no ácida.
Ante la presencia de una alta sospecha clínica es necesaria la
realización de una biopsia intestinal para confirmar el diag- Pregunta 60.- R: 2
nóstico. Se hará, al menos en una ocasión, estando el paciente La criptorquidia (descenso testicular incompleto) es el trastorno de la
consumiendo gluten. Una vez asegurado el diagnóstico se ini- diferenciación sexual más frecuente en los varones. Se estima que
ciará el tratamiento a base de una dieta rigurosamente exenta un 4,5 % de los niños tienen criptorquidia al nacimiento. Como el
de gluten de por vida. descenso testicular se produce en la última fase de la gestación,
La biopsia intestinal en el caso de la enfermedad celíaca no es pa- es más frecuente en prematuros. La mayoría de los testículos no
tognomónica, pero sí muy característica: aparece un infiltrado descendidos bajan al escroto espontáneamente en los primeros
inflamatorio en la lámina propia, junto con hiperplasia de las 3 meses de vida. Si esto no ha ocurrido a los 6 meses de edad, es
criptas y atrofia vellositaria. poco probable que lo haga más adelante. Las consecuencias de
El seguimiento se realizará valorando la evolución clínica y la criptorquidia permanente son:
determinando periódicamente los anticuerpos para evaluar
el cumplimiento de la dieta. Los anticuerpos más sensibles • Riesgo de esterilidad en los casos bilaterales.
y específicos son los Ig A antitransglutaminasa (respuesta 2 • Riesgo de degeneración maligna a seminoma.
correcta). • Hernias asociadas.
• Torsión testicular en el lado afecto.
Pregunta 58.- R: 2 • Efectos psicológicos.
Estamos ante un cuadro de atresia de vías biliares extrahepáticas; lo
típico es un cuadro de colestasis en un lactante de 2-3 semanas La edad idónea para realizar el tratamiento es entre los 9 y los 15
(predominio de bilirrubina directa, coluria, acolia). meses de edad. Consiste en el descenso y la fijación quirúrgica
La prueba diagnostica sería la realización de un HIDA, donde se del testículo al escroto (orquidopexia); en todo caso es primordial
aprecia captación del contraste por el hígado, pero que no llega realizarla antes de la pubertad, sobre todo para eliminar el riesgo
al intestino (ausencia de excreción). de degeneración maligna.
El tratamiento, aparte de las medidas de mantenimiento, consiste
en la realización de una hepatoportoenterostomía o técnica de Pregunta 61.- R: 4
Kasai, para intentar restablecer el flujo biliar y disminuir el daño En un lactante que presenta un cuadro clínico inespecífico (vómitos,
hepático. En la mayoría de los casos, el tratamiento definitivo es febrícula y pérdida de apetito), junto con leucocitosis, leucocituria
el trasplante hepático. y nitritos positivos en orina, debemos sospechar una infección
del tracto urinario. Será necesario realizar un cultivo de orina, en
este caso obtenido mediante punción suprapúbica, e iniciar el
tratamiento antibiótico.

Pregunta 62.- R: 3
Respecto al RVU, hay que saber que es la anomalía congénita más
frecuente de la unión ureterovesical. Se suele diagnosticar al
estudiar las ITUs.
Se clasifican según la intensidad, el grado de dilatación ureteral y la
deformidad calicial (grados I-V y reflujo intrarrenal).
La principal complicación es la nefropatía por reflujo intrarrenal, que
es la causa de hasta un 20% de las insuficiencias renales y la 1ª
causa de HTA en la infancia
Para el diagnóstico se realiza una cistografía miccional. Se deben
descartar malformaciones anatómicas, evaluar el tamaño renal y
descartar cicatrices con el DMSA.
El tratamiento quirúrgico se establece en función del grado de reflujo.
Habitualmente los reflujos grados I y II se resuelven espontánea-
mente, en el grado III un 50% precisa tratamiento por vía endos-
cópica, y los del grado IV y V precisarán tratamiento quirúrgico.

Pregunta 63.- R: 4
El síndrome hemolítico urémico (SHU) suele presentarse en niños
menores de 4 años. En los días previos el niño tiene una GEA
Pregunta 58. Colestasis neonatal. enteroinvasiva (el agente más frecuente es el E. coli O-156). El

CTO Medicina • C/Nuñez de Balboa 115 • 28006 Madrid • Tfno. (0034) 91 782 43 30/33/34 • E-mail: secretaria@ctomedicina.com • www. ctomedicina.com 10

Willy Aguilar
Comentarios de Test a distancia 1ª vuelta
Pediatría
los de células gliales (astrocitoma, ependimoma, glioblastoma
patógeno produce unas toxinas que favorecen por la lesión a nivel
multiforme), y los tumores neuroectodérmicos primitivos (medu-
endotelial la agregación plaquetaria y la formación de trombos y
loblastoma, pinealoblastoma), siendo los primeros más frecuentes
secundariamente una anemia hemolítica microangiopática.
globalmente, aunque la incidencia depende de la edad.
El órgano más afectado en este cuadro es el riñón produciendo un
cuadro de IRA. Otros en menor frecuencia son: intestino, SNC,...
El diagnóstico se basa en: Pregunta 67.- R: 3
• Anemia hemolítica microangiopática: haptoglobina descendida El cuadro que se expone es una niña con un neuroblastoma. La edad
con esquistocitos en el frotis de sangre periférica. de aparición típica es en menores de 2 años. Se suele localizar en
• Plaquetopenia. el abdomen a nivel de las glándulas suprarrenales y la clínica más
• IRA: por ecografía se debe diferenciar de la trombosis venosa frecuente es la masa abdominal que cruza línea media. Entre los
bilateral. síndromes paraneoplásicos (no modifican el pronóstico) figura el
opsoclonus-mioclonus (caso clínico) y la diarrea secretora. Para
el diagnóstico se utiliza la TC craneal, catecolaminas en orina
Pregunta 64.- R: 2
aumentadas y la gammagrafía con MIBG. El tratamiento depende
LEstamos ante un cuadro de torsión testicular. Se debe realizar diag-
del estadio, realizándose cirugía, quimioterapia y radioterapia. La
nostico diferencial con la orquioepididimitis.
supervivencia es del 50%.
La clínica consiste en un dolor agudo intenso, con tumefacción
escrotal, sin fiebre ni traumatismo previo. El reflejo cremastérico
suele estar anulado. Pregunta 68.- R: 4
El diagnóstico se basa fundamentalmente en la clínica, en caso de El tumor de Wilms se asocia con anomalías genitourinarias, hemihiper-
duda se puede recurrir a la gammagrafía de flujo testicular o a la trofia y aniridia. Se han encontrado deleciones en el cromosoma
realización de una eco-Doppler. 11. La edad al diagnóstico es en niñas menores de tres años. La
El tratamiento consiste en la reducción manual o fijación quirúrgica clínica más frecuente es la masa abdominal asintomático. Otros
del teste afecto y del contralateral. El tiempo de viabilidad del como HTA, hematuria, etc. La localización más frecuente de las
testículo torsionado es de 4–6 horas, por lo que siempre se debe metástasis es el pulmón. El diagnóstico se basa en la TC y la PAAF,
considerar una urgencia quirúrgica. no aconsejándose la realización de biopsia.

Torsión Orquioepididimitis
Pregunta 69.- R: 5
Ante un niño que debuta de forma brusca con un síndrome constitucio-
MC Dolor brusco, intenso Más progresivo nal junto con pancitopenia y hepatoesplenomegalia, el diagnóstico
Edad 10-20 Mayores más probable será el de leucemia aguda (respuesta 5)..
Episodios parecidos Prostatismo, cirugía baja,
Antc. personales
previos sondajes, parotiditis Pregunta 70.- R: 4
Escroto Inflamado? Inflamado El tumor que tiene tendencia a presentar calcificaciones intraneoplá-
sicas (hasta en un 80% de los casos) es el neuroblastoma y no el
Doloroso, elevado y tumor de Wilms (respuesta falsa 4).
Teste Doloroso, móvil
horizontalizado
Con las pruebas de imagen se puede confirmar que el tumor de
Prehn No? Sí Wilms (nefroblastoma) es de origen intrarrenal. En ocasiones,
Hidrocele Sí Más frecuente los pacientes con nefroblastoma pueden presentar policitemia
debido a la producción de eritropoyetina.
Reflejo cremastérico Anulado Permanece
Las metástasis pulmonares son las más frecuentes dentro del tumor
Menos flujo que el Igual o mayor que el con- de Wilms, pero su presencia no suele contraindicar la cirugía del
Eco Doppler
contralateral tralateral
tumor.
Pregunta 64. Patología testicular aguda. El tumor de Wilms puede ser bilateral, sobre todo en las formas
familiares (5%).
Pregunta 65.- R: 3
Caso clínico de PTI (púrpura trombótica idiomática). Típicamente apa- Pregunta 71.- R: 4
rece como un cuadro de petequias generalizadas, con disminución Estamos ante un cuadro de sarampión. El caso clínico va relatando
del número de plaquetas, tras un catarro de vías altas. No suele detalladamente las fases de esta enfermedad: Fase prodrómica:
existir anemia ni otros signos de diátesis hemorrágica. El pronóstico catarro con tos y fiebre, manchas de Koplik (lesiones blanque-
es bueno evolucionando hacia la recuperación espontánea en la cinas sobre base eritematosa en mucosa subyugal), es un dato
mayoría de los casos (respuesta cierta: 3). patognomónico de esta enfermedad. Fase exantemática: inicio del
exantema por la cara y descenso. El exantema es maculopapular, no
Pregunta 66.- R: 4 pruriginoso. La temperatura sube hasta 39º-40º. Fase de resolución
El cáncer pediátrico difiere del cáncer en el adulto en su naturaleza, :el exantema desaparece en el mismo orden que apareció.
distribución y pronóstico. Su tratamiento es difícil debido a los Entre las complicaciones figuran:
efectos adversos que la cirugía, la quimioterapia, y la radioterapia • Cuadros ORL: complicación más frecuente.
pueden tener sobre el crecimiento. La leucemia linfoide aguda es • Neumonías: en los niños es más frecuente la sobreinfección bac-
el más frecuente, aumentando su incidencia en el síndrome de teriana, más que la neumonía de células gigantes de Hecht.
Down y de Bloom, así como en inmunodeficiencias. Los linfomas • Afectación del SNC: encefalitis aguda y PEES (panencefalitis es-
siguen a las leucemias en frecuencia, salvando el grupo de edad clerosante subaguda): forma de encefalitis por virus lentos, con
de entre 10 y 14 años, en que superan a las anteriores. El neuro- un pronóstico pésimo, el diagnostico se hace detectando en LCR
blastoma, es más frecuente en varones y en la raza blanca, y el aumento de Acs frente al sarampión.
90% se diagnostican por debajo de los 5 años de edad. En cuanto
a los tumores óseos, en niños mayores y adolescentes, predomina Pregunta 72.- R: 3
el osteosarcoma, seguido del Ewing, que es el más frecuente en Con los datos de febrícula y adenopatías retroauriculares y cervicales
menores de10 años. Dentro de los tumores cerebrales, los más dolorosas debemos pensar en una rubéola.
frecuentes después de las neoplasias hematológicas, se distinguen El período de máxima transmisión abarca desde 7 días antes de

CTO Medicina • C/Nuñez de Balboa 115 • 28006 Madrid • Tfno. (0034) 91 782 43 30/33/34 • E-mail: secretaria@ctomedicina.com • www. ctomedicina.com 11

Willy Aguilar
Comentarios de Test a distancia 1ª vuelta
Pediatría
aparición del exantema hasta 7 u 8 días después de que éste Se caracteriza por una primera fase de fiebre alta, con buen estado
haya aparecido. general, que dura de 3 a 4 días. Posteriormente, en una segunda
La rubéola suele darse en niños mayores de 6 meses, pues antes fase, aparece bruscamente un exantema maculopapuloso poco
los anticuerpos maternos actúan como protectores (respuesta confluente que afecta al tórax, abdomen y raíz de miembros.
3). Una vez superada la enfermedad suele quedar inmunidad Son datos típicos del hemograma la presencia de leucocitosis con
permanente. neutrofilia en las primeras 24-36 horas de evolución, por lo que
en un primer momento puede ser difícil establecer el diagnóstico
Pregunta 73.- R: 1 de infección vírica (respuesta 2).
La pancitopenia puede ser congénita o adquirida. La más frecuente
es esta última, y las causas más importantes son los fármacos, los Pregunta 77.- R: 4
tóxicos, las infecciones, radiaciones y procesos inmunitarios. Las Recordemos los criterios diagnósticos de la enfermedad de Kawasaki
leucemias y la hemoglobinuria paroxística nocturna (HPN) son o sd. mucocutáneo – ganglionar.
etiologías más raras.
En este caso clínico, no podemos sospechar que un fármaco sea la A. Fiebre de al menos 5 días de duración
causa, porque no tenemos datos de que el niño haya tomado Presencia de 4 de los cinco siguientes signos:
ningún medicamento previamente (probablemente haya reci- 1. Conjuntivitis bilateral no purulenta.
bido antitérmicos, pero es raro que provoquen esta reacción). 2. Alteraciones en la mucosa de la orofaringe, con inyección faríngea;
Tampoco se señala que el niño tenga orinas anormales, y por labios secos con fisuras, inyectados o ambos, y lengua “”en fresa””
B.
otro lado la HPN es excepcional en la infancia. La pancitopenia 3. Alteraciones en las zonas periféricas de las extremidades, como ede-
no se asocia en general a los linfomas antes del tratamiento. Las ma y eritema en manos o pies, descamación de inicio periungueal.
4. Exantema, de inicio en el tronco; polimorfo, no vesicular.
leucemias afortunadamente son mucho menos frecuentes que
5. Linfadenopatía cervical unilateral.
las infecciones víricas.
El cuadro clínico descrito, es el típico de mononucleosis infecciosa. El C. La enfermedad no se puede explicar por ninguna otra causa conocida.
virus Epstein Barr, los virus A y B de la hepatitis, el parvovirus B19, Pregunta 39. Criterios diagnósticos de la enfermedad de Kawasaki.
el VIH y el citomegalovirus, son los más asociados a pancitopenia.
Para confirmar el diagnóstico habría que pedir serología específica, Además de estos síntomas se puede encontrar patología muy varia-
aunque el tratamiento es sintomático, y en cuanto a la pancitope- da: artritis , pericarditis, iritis,... Es muy llamativa la trombocitosis,
nia, actitud expectante, pues se resuelve espontáneamente en la especialmente en la fase subaguda de la enfermedad.
mayoría de las ocasiones. El tratamiento consiste en el empleo de AAS y Gammaglobulina en la
fase aguda, manteniendo el AAS en dosis antiagregantes posterior-
Pregunta 74.- R: 1 mente. El empleo de la gammaglobulina en la fase aguda previene
El caso que se nos presenta es una varicela. Ya sabemos que el agente el desarrollo de aneurismas coronarios en la fase subaguda.
causal es el virus de la varicela–zoster, de la familia de los herpes
virus. La fase prodrómica consiste en un catarro de vías altas con Pregunta 78.- R: 2
febrícula. Lo característico del exantema es que suele afectar a Este paciente presenta una púrpura de Schölein-Henoch. Como ya
tronco y parte proximal de extremidades, respetando las zonas recordáis, es una vasculitis de pequeño vaso.
distales. Las lesiones aparecen en distintos estadios evolutivos (o La fase prodrómica suele consistir en proceso ORL. Posteriormente
lesiones en cielo estrellado); estas son lesiones eritematosas con puede aparecer, en orden de frecuencia:
vesículas y otras en fase de costra. Las lesiones pican mucho. La • Afectación cutánea: púrpura palpable, en mmii, que suele cursar
complicación más frecuente es la sobreinfección bacteriana de en brotes.
las lesiones. • Artritis de rodillas y tobillos.
Otras complicaciones son: • Dolor abdominal cólico, vómitos y deposiciones con sangre.
• Neumonía varicelosa: en la infancia es más frecuente la secundaria • Afectación renal: hematuria con o sin proteinuria. Este es el factor
a sobreinfección bacteriana, más que la producida por el propio que marca el pronóstico.
virus. • Sólo en menos de un 1% se produce afectación del SNC.
• Afectación del SNC: lo más frecuente es en forma de cerebelitis, El tratamiento consiste en antiinflamatorios para las molestias
que evoluciona de forma favorable. articulares y en ciclos cortos de corticoides, en caso de dolor
abdominal intenso.
La varicela en los niños sanos no precisa tratamiento con aciclovir. Son
indicaciones para su uso: neonatos e inmunodeprimidos. Pregunta 79.- R: 2
Para el diagnóstico de mononucleosis infecciosa, aparte de la sospecha
Pregunta 75.- R: 3 clínica, podemos encontrar:
La pregunta hace referencia al llamado eritema infeccioso, megaloeri- • Leucocitosis (10000-20000) con más de un 20-40% de linfocitos
tema o 5ª enfermedad. En el MIR lo suelen preguntar para confun- atípicos.
dirlo con el exantema súbito, roseola infantil o 6ª enfermedad. • Aumento de las transaminasas.
El eritema infeccioso tiene como agente causal el Parvovirus B 19. El • Acs heterófilos: son específicos frente al VEB. Es útil en mayores
cuadro se caracteriza por febrícula y aparición de un exantema en de 4 años; por debajo de esta edad, la sensibilidad es < del 20%.
encaje o cartográfico en mejillas y zona proximal de extremidades. Puede permanecer positivo hasta 9 meses después, por lo que no
Se resuelve espontáneamente, pero en ocasiones puede recidivar es útil para el diagnóstico de infección activa.
cuando el niño se estresa, tiene fiebre,... • Acs específicos contra el VEB:
Entre las complicaciones figuran la aplasia medular en niños suscep- - Fase aguda:
tibles (con enfermedades hematológicas). IgM + - IgG: frente a la cápside viral : CVA
EA + (Ag temprano)
Pregunta 76.- R: 2
Caso clínico de exantema súbito (roséola infantil o 6ª enfermedad). EL VEB se ha asociado con distintos tumores como el linfoma Burkitt,
Su agente es el Herpes virus tipo 6 y afecta principalmente a niños carcinoma nasofaríngeo, sd. de Duncan y diversos sds. linfopro-
menores de 2 años. liferativos.

CTO Medicina • C/Nuñez de Balboa 115 • 28006 Madrid • Tfno. (0034) 91 782 43 30/33/34 • E-mail: secretaria@ctomedicina.com • www. ctomedicina.com 12

Willy Aguilar
Comentarios de Test a distancia 1ª vuelta
Pediatría
El tratamiento consiste en reposo; en algunos casos puede ser útil • Embarazo.
la administración de corticoides (obstrucción de la vía aérea alta, • La alergia al huevo ha dejado de ser una contraindicación abso-
anemia hemolítica autoinmune, convulsiones,...). luta.

Pregunta 80.- R: 4 Pregunta 84.- R: 4


El cuadro del que vamos a hablar a continuación es de una tos ferina. Existen actualmente dos tipos de vacunas: Polio oral (Sabin) y Polio
La edad con más riesgo es el periodo de lactante (la madre no le im (Salk).
da Acs protectores, porque estos van disminuyendo a lo largo La polio Sabin es una vacuna de virus vivos atenuados que produce
de la vida). inmunidad local tipo IgA y vacunación comunitaria por contacto
La clínica consta de una fase prodrómica: fase catarral, con febrícula, fecal-oral. Como efecto adverso más importante destacan algunos
rinorrea y tos blanda. Este es el periodo de máxima contagiosidad. casos aislados de polio en niños sanos, lo que ha motivado en
La fase paroxística: consiste en episodios de golpes o accesos los últimos años que se estén potenciando las vacunas de virus
de tos, seguido en muchas ocasiones de vómitos. En los niños inactivados.
mayores es característico el “ gallo” inspiratorio. Y por último, la Está contraindicada en personas inmunodeprimidas y personas que
fase de convalecencia: esta fase se caracteriza por encontrarse conviven con éstos.
el niño afebril. La vacuna tipo Salk (im) está elaborada con virus inactivados, es me-
El diagnóstico se basa en el cultivo del moco nasal en medio de nos eficaz, pero más segura. Es la que actualmente recomienda
Bordet-Gengou. la AEP. La pauta de administración es a los 2-4-6 –18 meses y a
En la analítica es típico encontrar una linfocitosis absoluta. Para el los 4 años.
tratamiento se utiliza la eritromicina.
Pregunta 85.- R: 5
Pregunta 81.- R: 5 Lo básico que tenéis que recordar de la vacuna DTP es lo siguiente:
Respecto al SIDA en Pediatría, hay que recordar: • Componentes:
- Difteria: toxoide.
• La principal vía de transmisión es vertical, durante el parto. El - Tétanos: toxoide.
porcentaje de infectados es aproximadamente del 5-8%. - Pertussis: dos tipos: celular y acelular. Esta última con muchos
• El periodo de latencia es menor que en adulto. Se subdivide en menos efectos adversos a nivel neurológico. Recomendada por
SIDA precoz (periodo de latencia de pocos meses, clínica antes la AEP. La Pertussis celular estaba contraindicada en mayores
del año de vida, predominando las manifestaciones neurológicas, de 7 años, porque a partir de esa edad hay más riesgos que
pronostico malo) y tardío (más parecido al del adulto, predomi- beneficios.
nando la clínica infecciosa). • Indicaciones: en el calendario vacunal se administra a los 2,4,6 y
• Las manifestaciones clínicas más frecuentes en pediatría, compa- 18 meses y a los 4 años se puede administrar DT o DTPa , luego
rándolo con los adultos son: cuadros ORL, parotiditis, neumonía cada 10 años dT. Las primeras dosis se pueden administrar en un
intersticial linfoide y manifestaciones neurológicas (calcificaciones mismo preparado con otras vacunas (polio inactivada, Hib, VHB),
de los ganglios basales, atrofia cerebral,...). La causa más frecuente para minimizar el número de pinchazos.
de muerte es la neumonía por P. Carinii. En pacientes inmunodeprimidos se puede administrar sin riesgo.
• Son infecciones menos frecuentes en los niños: TBC, hepatitis B,
linfomas y sarcoma de Kaposi. Pregunta 86.- R: 4
La vacuna del Hib es una vacuna conjugada, muy segura y eficaz.
Pregunta 82.- R: 1 Protege frente a infecciones invasivas (meningitis y epiglotitis).
Es importante realizar quimioprofilaxis con AZT durante el embarazo, En el calendario vacunal se incluye a los 2-4-6 y 18 meses. Estaría
el momento del parto y luego al RN. indicada solamente en niños menores de 5 años.
El tratamiento se basa en la triple terapia, al igual que en los adul-
tos. Pregunta 87.- R: 4
Para evaluar la evolución, se emplea la carga viral y el cociente CD4/ La vacuna frente a la hepatitis B está realizada por ingeniería gené-
CD8, recordando que la cifra de linfocitos hay que extrapolarla tica. Produce en las personas que la recibe Acs frente al Ag s. En
según la edad del niño. el calendario vacunal se puede administrar con diversas pautas:
(0-1-6), (0,2,6) y (2, 4,6) y una cuarta a los 18 meses. Para prevenir la
< de 18 meses > 18 meses trasmisión de la hepatitis B en el neonato, en el caso que la madre
• Dos resultados posi- sea portadora, se debe asociar con gammaglobulina especifica
tivos: al nacimiento.
+ PCR Está indicada a cualquier edad si el niño no ha sido vacunado previa-
• ELISA+ y se confirma por Western Blot.
+ Cultivo viral
• O los criterios del apartado anterior.
mente. Sus contraindicaciones son muy escasas. Recordar que el
+ Agp 24 embarazo no es una contraindicación absoluta, pero se aconseja
evitar todo tipo de vacunas siempre que sea posible.
• O clínica del SIDA.
Pregunta 82. Diagnóstico de SIDA en la infancia. Pregunta 88.- R: 4
Quimioprofilaxis frente a los agentes más frecuentes de meningitis.
Pregunta 83.- R: 3 • Meningococo: a todos los contactos, independiente de la edad,
La triple vírica protege contra el virus del sarampión, rubéola y paro- con rifampicina en dosis de 10 mg/K/dosis cada 12 horas 4 dosis
tiditis. Es una vacuna de virus vivos atenuados. +/- vacuna en caso del meningococo C.
Indicaciones: a los 15 meses y una dosis de recuerdo a los 4 años, a • Hib: niños menores de 5 años sin vacunar y adultos que conviven
niños y niñas. con menores de 5 años sin vacunar (éstos pueden ser portadores):
Efectos adversos: cuadro de fiebre y exantema, más atenuados que rifampicina en dosis de 20 mg/K/dosis cada 24 horas 4 dosis +7-
las infecciones frente a las que protege (son virus vivos). vacuna.
Contraindicaciones: • Neumococo:noexistequimioprofilaxisactiva.Sepuedeindicarlavacuna
• Inmunodeprimidos: excepto niño HIV+. del neumococo de 7 serotipos para prevenir futuros contactos.

CTO Medicina • C/Nuñez de Balboa 115 • 28006 Madrid • Tfno. (0034) 91 782 43 30/33/34 • E-mail: secretaria@ctomedicina.com • www. ctomedicina.com 13

Willy Aguilar
Comentarios de Test a distancia 1ª vuelta
Pediatría
ECG suele ser de características normales para un RN. El diagnóstico
Pregunta 89.- R: 2 se confirma mediante la ecografía.
Vamos a esquematizar a continuación la actitud que hay que llevar El tratamiento consiste en la perfusión de PGE de forma urgente
a cabo con un niño que ha tenido un contacto con un bacilífero para mantener abierto el ductus, y cuanto antes realizar la co-
positivo. rrección quirúrgica. La técnica de elección es el switch arterial o
Jatene, que consiste en seccionar la salida de los grandes vasos
e intercambiarlos.

Pregunta 93.- R: 2
Los malos tratos en la infancia se han producido a lo largo de la
historia de toda la humanidad y en todas las sociedades. Es muy
importante detectarlos precozmente pues hay casos fatales que
producen daños importantes al niño, incluso la muerte. La actitud
de los padres (despreocupación por la gravedad de las lesiones,
justificaciones poco lógicas para las lesiones detectadas, preocu-
pación por la propia situación personal y no por la del niño,…),
la actitud del niño (ansiedad excesiva al hablarle y explorarle,
pasividad inadecuada para su edad, conducta agresiva, distancia
o proximidad excesivas con los padres), y la exploración física
(lesiones traumáticas en distintas fases de evolución, lesiones
en localizaciones que no corresponden con la actividad del niño
ni su desarrollo psicomotor, fracturas raras en niños como la del
acromion, …), son las herramientas que tenemos para sospechar
el diagnóstico. En tal caso, hay que ponerse en contacto con los
Servicios Sociales y de Protección y Atención al Menor corres-
pondientes, y ampliar el estudio con una serie ósea completa, un
Pregunta 90.- R: 3 examen del fondo de ojo, un TAC craneal, tomando fotografías en
El SMSL, muerte repentina e inexplicable de un niño menor de un caso de lesiones cutáneas.
año, supone la causa principal de muerte entre el mes de vida
y el año de edad en los países desarrollados. Este síndrome se Pregunta 94.- R: 3
correlaciona con la posición de decúbito prono y lateral para La CoA es más frecuente en niñas con sd de Turner. El tipo más fre-
dormir. Tras este factor posicional, el tabaquismo materno es cuente es yuxtaductal. Los niños suelen estar asintomáticos. En
actualmente el principal factor de riesgo del SMSL. Otros factores la exploración es típico encontrar un soplo sistodiastólico interes-
son la prematuridad, el antecedente de hermano muerto por SMSL, capular y pulsos femorales débiles con HTA en mmss. En la Rx de
sexo masculino, etc. tórax aparecen escotaduras costales en los bordes inferiores de
Debido a la ausencia de tratamiento es muy importante la prevención las costillas y a veces el llamado signo del 3. En el ECG aparecen
con medidas como la posición de decúbito supino para dormir y signos de hipertrofia de VI. El tratamiento se realiza mediante
la lactancia materna (que se considera el factor preventivo más arteriografía con dilatación con balón o mediante cirugía colo-
importante). cando un parche.

Pregunta 91.- R: 5 Pregunta 95.- R: 2


El ductus arterioso persistente es una cardiopatía típica de RNPT, en Pregunta sobre el desarrollo puberal normal. Recuerda que el aumento
los que la síntesis local de PGE1 mantiene el ductus abierto en la del tamaño testicular en el varón es el primer signo de pubertad
vida extrauterina, unido a la sobrecarga de volumen. (respuesta 2). En las niñas, el primer dato a la exploración sugestivo
Esta cardiopatía congénita también se asocia a la rubéola congé- de inicio de la pubertad, es la aparición de botón mamario.
nita.
La clínica consiste en un soplo continuo, en maquinaria o de Gibson, Pregunta 96.- R: 3
más audible en 2º espacio intercostal izquierdo; los pulsos femorales La telarquia prematura es el desarrollo mamario prematuro en niñas
son saltones. El niño puede presentar clínica de ICC. sin otros caracteres sexuales secundarios, sin aceleración de la
En la Rx de tórax se aprecia plétora pulmonar con aumento de ca- velocidad de crecimiento ni de la edad ósea. La etiopatogenia no
vidades izquierdas. El diagnóstico definitivo lo obtenemos por está clara, y es probable que sea multifactorial. Es más frecuente
ecografía. en los 2 primeros años de vida, y en general tiene buen pronóstico,
En los RNPT, si no existen contraindicaciones, se comienza con la con regresión espontánea en la mayoría de los casos en 2-3 años.
administración de indometacina, y si no se soluciona, tratamiento Sin embargo, a veces es persistente, pudiendo evolucionar a una
quirúrgico. En caso de no ser un RNPT, el tratamiento de entrada pubertad precoz o adelantada, sin que se conozcan actualmente
es quirúrgico. los factores de riesgo para ello. Para el diagnóstico son útiles:
• Rx de mano y muñeca izquierdas (edad ósea).
Pregunta 92.- R: 4 • Ecografía pélvica.
La TGA es la cardiopatía cianógena más frecuente de inicio en el • Niveles basales de estradiol.
RNPT (recordad que la c. cianógena más frecuente en conjunto • La prueba diagnóstica para excluir una pubertad precoz es el test
es la T. De Fallot). de GnRH (respuesta de FSH y LH ante el estímulo con análogos de
LA TGA más frecuente es la que cursa con un tabique interventricular GnRH).
íntegro.
La clínica suele consistir en un RN que en las primeras horas de vida Pregunta 97.- R: 4
presenta taquipnea y cianosis intensa (conforme se va cerrando el Existen numerosos síndromes genéticos y polimalformativos que se
ductus), no tiene soplo y los pulsos son débiles. En la Rx de tórax manifiestan como talla baja. Sin embargo, el síndrome de Klinefelter
se aprecia un pedículo cardíaco estrecho con plétora pulmonar. El se caracteriza por presentar talla alta junto con otras alteraciones

CTO Medicina • C/Nuñez de Balboa 115 • 28006 Madrid • Tfno. (0034) 91 782 43 30/33/34 • E-mail: secretaria@ctomedicina.com • www. ctomedicina.com 14

Willy Aguilar
Comentarios de Test a distancia 1ª vuelta
Pediatría
(retraso mental, DM, alteraciones de la función tiroidea, etc).
Recuerda que el síndrome de Silver-Russell se trata de un cuadro de
talla baja que se manifiesta desde la etapa fetal.

Pregunta 98.- R: 1
La causa más frecuente de convulsiones en el RN es la encefalopatía
hipóxico-isquémica. Las crisis suelen ser focales, no generali-
zadas, puesto que el SNC del neonato no está completamente
mielinizado. El diagnóstico se basa en el EEG. El fenobarbital es
el fármaco de primera elección. El pronóstico dependerá de la
causa desencadenante.

Pregunta 99.- R: 3
La enuresis es la emisión persistente de orina, en la ropa o en la
cama, voluntaria o involuntaria, después de los 5 años de edad.
Su prevalencia es mayor en niños que en niñas. Hay un claro
patrón familiar y el 90% de los casos son primarios o persistentes
(el niño no ha llegado nunca a tener un control completo de la
orina). Los casos secundarios son más frecuentes entre los 5 y los
8 años de edad, y característicamente son transitorios y de buen
pronóstico. La enuresis diurna es más frecuente en niñas, y casi
siempre se debe al aplazamiento de la micción hasta revasar la
capacidad de la vejiga. El tratamiento más correcto debe buscarse
después de una evaluación psicosocial y la exploración física. Lo
más efectivo son las técnicas conductuales. La imipramina logra
un 50% de éxitos, con un 30% de recaidas tras su retirada. La
desmopresina es efectiva, pero asimismo, tiene una alta tasa de
recaídas, y supone un riesgo de retención hídrica e hiponatremia,
por lo que se recomienda sólo en situaciones puntuales en que se
desee un control de la orina.

Pregunta 100.- R: 2
El cuadro clínico hace referencia a un niño con una convulsión
febril. La clínica típica consiste en fiebre alta, crisis tónico-clónica
generalizada de breve duración con un periodo postcrítico corto. La
edad de presentación es entre los 6 meses y 5 años. Existen antece-
dentes familiares hasta en un 30% de los casos. El tratamiento en la
fase aguda consiste en la administración de diacepam intrarrectal y
medidas para disminuir la hipertermia.

CTO Medicina • C/Nuñez de Balboa 115 • 28006 Madrid • Tfno. (0034) 91 782 43 30/33/34 • E-mail: secretaria@ctomedicina.com • www. ctomedicina.com 15

Willy Aguilar
CTO Medicina • C/Nuñez de Balboa 115 • 28006 Madrid • Tfno. (0034) 91 782 43 30/33/34 • E-mail: secretaria@ctomedicina.com • www. ctomedicina.com 16

Willy Aguilar

Das könnte Ihnen auch gefallen